Свойства арккосинус: Арксинус, арккосинус, арктангенс, арккотангенс числа. Функции и их свойства. Курсы по математике

Содержание

Arctg 2 чему равен. Арксинус, арккосинус

Функции sin, cos, tg и ctg всегда сопровождаются арксинусом, арккосинусом, арктангенсом и арккотангенсом. Одно является следствием другого, а пары функций одинаково важны для работы с тригонометрическими выражениями.

Рассмотрим рисунок единичной окружности, на котором графически отображено значений тригонометрических функций.

Если вычислить arcs OA, arcos OC, arctg DE и arcctg MK, то все они будут равны значению угла α. Формулы, приведенные ниже, отражают взаимосвязь основных тригонометрических функций и соответствующих им арков.

Чтобы больше понять о свойствах арксинуса, необходимо рассмотреть его функцию. График имеет вид асимметричной кривой, проходящей через центр координат.

Свойства арксинуса:

Если сопоставить графики sin и arcsin , у двух тригонометрических функций можно найти общие закономерности.

Арккосинус

Arccos числа а — это значение угла α, косинус которого равен а.

Кривая y = arcos x зеркально отображает график arcsin x, с той лишь разницей, что проходит через точку π/2 на оси OY.

Рассмотрим функцию арккосинуса более подробно:

  1. Функция определена на отрезке [-1; 1].
  2. ОДЗ для arccos — .
  3. График целиком расположен в I и II четвертях, а сама функция не является ни четной, ни нечетной.
  4. Y = 0 при x = 1.
  5. Кривая убывает на всей своей протяженности. Некоторые свойства арккосинуса совпадают с функцией косинуса.

Некоторые свойства арккосинуса совпадают с функцией косинуса.

Возможно, школьникам покажется излишним такое «подробное» изучение «арков». Однако, в противном случае, некоторые элементарные типовые задания ЕГЭ могут ввести учащихся в тупик.

Задание 1. Укажите функции изображенные на рисунке.

Ответ:

рис. 1 – 4, рис.2 — 1.

В данном примере упор сделан на мелочах. Обычно ученики очень невнимательно относятся к построению графиков и внешнему виду функций. Действительно, зачем запоминать вид кривой, если ее всегда можно построить по расчетным точкам. Не стоит забывать, что в условиях теста время, затраченное на рисунок для простого задания, потребуется для решения более сложных заданий.

Арктангенс

Arctg числа a – это такое значение угла α, что его тангенс равен а.

Если рассмотреть график арктангенса, можно выделить следующие свойства:

  1. График бесконечен и определен на промежутке (- ∞; + ∞).
  2. Арктангенс нечетная функция, следовательно, arctg (- x) = — arctg x.
  3. Y = 0 при x = 0.
  4. Кривая возрастает на всей области определения.

Приведем краткий сравнительный анализ tg x и arctg x в виде таблицы.

Арккотангенс

Arcctg числа a — принимает такое значение α из интервала (0; π), что его котангенс равен а.

Свойства функции арккотангенса:

  1. Интервал определения функции – бесконечность.
  2. Область допустимых значений – промежуток (0; π).
  3. F(x) не является ни четной, ни нечетной.
  4. На всем своем протяжении график функции убывает.

Сопоставить ctg x и arctg x очень просто, нужно лишь сделать два рисунка и описать поведение кривых.

Задание 2. Соотнести график и форму записи функции.

Если рассуждать логически, из графиков видно, что обе функции возрастающие. Следовательно, оба рисунка отображают некую функцию arctg. Из свойств арктангенса известно, что y=0 при x = 0,

Ответ: рис. 1 – 1, рис. 2 – 4.

Тригонометрические тождества arcsin, arcos, arctg и arcctg

Ранее нами уже была выявлена взаимосвязь между арками и основными функциями тригонометрии. Данная зависимость может быть выражена рядом формул, позволяющих выразить, например, синус аргумента, через его арксинус, арккосинус или наоборот. Знание подобных тождеств бывает полезным при решении конкретных примеров.

Также существуют соотношения для arctg и arcctg:

Еще одна полезная пара формул, устанавливает значение для суммы значений arcsin и arcos, а также arcctg и arcctg одного и того же угла.

Примеры решения задач

Задания по тригонометрии можно условно разделить на четыре группы: вычислить числовое значение конкретного выражения, построить график данной функции, найти ее область определения или ОДЗ и выполнить аналитические преображения для решения примера.

При решении первого типа задач необходимо придерживаться следующего плана действий:

При работе с графиками функций главное – это знание их свойств и внешнего вида кривой. Для решения тригонометрических уравнений и неравенств необходимы таблицы тождеств. Чем больше формул помнит школьник, тем проще найти ответ задания.

Допустим в ЕГЭ необходимо найти ответ для уравнения типа:

Если правильно преобразовать выражение и привести к нужному виду, то решить его очень просто и быстро. Для начала, перенесем arcsin x в правую часть равенства.

Если вспомнить формулу arcsin (sin α) = α , то можно свести поиск ответов к решению системы из двух уравнений:

Ограничение на модель x возникло, опять таки из свойств arcsin: ОДЗ для x [-1; 1]. При а ≠0, часть сиcтемы представляет собой квадратное уравнение с корнями x1 = 1 и x2 = — 1/a. При a = 0, x будет равен 1.

Замечание 1

Таблица Брадиса — это таблица, позволяющая высчитывать значения арктангенсов и других тригонометрических функций с высокой точностью.

Для того чтобы воспользоваться таблицей Брадиса, ищут угол в градусах в крайнем столбце слева для синуса (для косинуса в соответствующем столбце справа), а затем в верхней строке минуты. На пересечении строки со столбцом находится искомое значение.

При необходимости найти значения обратных тригонометрических функций — таблицу Брадиса используют наоборот. Например, ищут числовое значение в таблице арктангенсов и тангенсов и для него определяют, в какой строке градусов и столбце минут оно находится.

Таким образом, Таблицу Брадиса можно использовать не только для поиска обычных тригонометрических функций, но и как таблицу арккосинуса и арксинуса, арктангенсов и арккотангенсов.

Сверху в этой статье расположена таблица значений arcsin и arccos, ближе к концу — таблица значений arctg и arcctg.

Таблица Брадиса: таблица arcsin, arccos, cos и sin

Рисунок 1. Таблица Брадиса таблица значений arcsin и arccos. Автор24 — интернет-биржа студенческих работ

Таблица значений арктангенсов и арккотангенсов, тангенсов и котангенсов

Рисунок 4. Таблица Брадиса: таблица значений арктангенсов arctg и арккотангенсов arctg. Автор24 — интернет-биржа студенческих работ

Определение и обозначения

Арксинус (y = arcsin x ) — это функция, обратная к синусу (x = sin y -1 ≤ x ≤ 1 и множество значений -π/2 ≤ y ≤ π/2 .
sin(arcsin x) = x ;
arcsin(sin x) = x .

Арксинус иногда обозначают так:
.

График функции арксинус

График функции y = arcsin x

График арксинуса получается из графика синуса, если поменять местами оси абсцисс и ординат. Чтобы устранить многозначность, область значений ограничивают интервалом , на котором функция монотонна. Такое определение называют главным значением арксинуса.

Арккосинус, arccos

Определение и обозначения

Арккосинус (y = arccos x ) — это функция, обратная к косинусу (x = cos y ).
Он имеет область определения -1 ≤ x ≤ 1 и множество значений 0 ≤ y ≤ π .
cos(arccos x) = x ;
arccos(cos x) = x .

Арккосинус иногда обозначают так:
.

График функции арккосинус


График функции y = arccos x

График арккосинуса получается из графика косинуса, если поменять местами оси абсцисс и ординат. Чтобы устранить многозначность, область значений ограничивают интервалом , на котором функция монотонна. Такое определение называют главным значением арккосинуса.

Четность

Функция арксинус является нечетной:
arcsin(- x) = arcsin(-sin arcsin x) = arcsin(sin(-arcsin x)) = — arcsin x

Функция арккосинус не является четной или нечетной:
arccos(- x) = arccos(-cos arccos x) = arccos(cos(π-arccos x)) = π — arccos x ≠ ± arccos x

Свойства — экстремумы, возрастание, убывание

Функции арксинус и арккосинус непрерывны на своей области определения (см. доказательство непрерывности).

Основные свойства арксинуса и арккосинуса представлены в таблице.

y = arcsin x y = arccos x
Область определения и непрерывность — 1 ≤ x ≤ 1 — 1 ≤ x ≤ 1
Область значений
Возрастание, убывание монотонно возрастает монотонно убывает
Максимумы
Минимумы
Нули, y = 0 x = 0 x = 1
Точки пересечения с осью ординат, x = 0 y = 0 y = π/2

Таблица арксинусов и арккосинусов

В данной таблице представлены значения арксинусов и арккосинусов, в градусах и радианах, при некоторых значениях аргумента.

x arcsin x arccos x
град.
рад. град. рад.
— 1 — 90° 180° π
— 60° 150°
— 45° 135°
— 30° 120°
0 0 90°
30° 60°
45° 45°
60° 30°
1 90° 0

≈ 0,7071067811865476
≈ 0,8660254037844386

Формулы

См. также: Вывод формул обратных тригонометрических функций

Формулы суммы и разности


при или

при и

при и


при или

при и

при и


при

при


при

при

Выражения через логарифм, комплексные числа

См. также: Вывод формул

Выражения через гиперболические функции

Производные

;
.
См. Вывод производных арксинуса и арккосинуса > > >

Производные высших порядков :
,
где — многочлен степени . Он определяется по формулам:
;
;
.

См. Вывод производных высших порядков арксинуса и арккосинуса > > >

Интегралы

Делаем подстановку x = sin t . Интегрируем по частям, учитывая что -π/2 ≤ t ≤ π/2 , cos t ≥ 0 :
.

Выразим арккосинус через арксинус:
.

Разложение в ряд

При |x| 1 имеет место следующее разложение:
;
.

Обратные функции

Обратными к арксинусу и арккосинусу являются синус и косинус , соответственно.

Следующие формулы справедливы на всей области определения:
sin(arcsin x) = x
cos(arccos x) = x .

Следующие формулы справедливы только на множестве значений арксинуса и арккосинуса:
arcsin(sin x) = x при
arccos(cos x) = x при .

Использованная литература:
И.Н. Бронштейн, К.А. Семендяев, Справочник по математике для инженеров и учащихся втузов, «Лань», 2009.

См. также:

Что такое арксинус, арккосинус? Что такое арктангенс, арккотангенс?


Внимание!
К этой теме имеются дополнительные
материалы в Особом разделе 555.
Для тех, кто сильно «не очень…»
И для тех, кто «очень даже…»)

К понятиям арксинус, арккосинус, арктангенс, арккотангенс учащийся народ относится с опаской. Не понимает он эти термины и, стало быть, не доверяет этой славной семейке.) А зря. Это очень простые понятия. Которые, между прочим, колоссально облегчают жизнь знающему человеку при решении тригонометрических уравнений!

Сомневаетесь насчёт простоты? Напрасно.) Прямо здесь и сейчас вы в этом убедитесь.

Разумеется, для понимания, неплохо бы знать, что такое синус, косинус, тангенс и котангенс. Да их табличные значения для некоторых углов… Хотя бы в самых общих чертах. Тогда и здесь проблем не будет.

Итак, удивляемся, но запоминаем: арксинус, арккосинус, арктангенс и арккотангенс — это просто какие-то углы. Ни больше ни меньше. Бывает угол, скажем 30°. А бывает угол arcsin0,4. Или arctg(-1,3). Всякие углы бывают.) Просто записать углы можно разными способами. Можно записать угол через градусы или радианы. А можно — через его синус, косинус, тангенс и котангенс…

Что означает выражение

arcsin 0,4 ?

Это угол, синус которого равен 0,4 ! Да-да. Это смысл арксинуса. Специально повторю: arcsin 0,4 — это угол, синус которого равен 0,4.

И всё.

Чтобы эта простая мысль сохранилась в голове надолго, я даже приведу разбивочку этого ужасного термина — арксинус:

arc sin 0,4
угол, синус которого равен 0,4

Как пишется, так и слышится.) Почти. Приставка arc означает дуга (слово арка знаете?), т. к. древние люди вместо углов использовали дуги, но это сути дела не меняет. Запомните эту элементарную расшифровку математического термина! Тем более, для арккосинуса, арктангенса и арккотангенса расшифровка отличается только названием функции.

Что такое arccos 0,8 ?
Это угол, косинус которого равен 0,8.

Что такое arctg(-1,3) ?
Это угол, тангенс которого равен -1,3.

Что такое arcctg 12 ?
Это угол, котангенс которого равен 12.

Такая элементарная расшифровка позволяет, кстати, избежать эпических ляпов.) Например, выражение arccos1,8 выглядит вполне солидно. Начинаем расшифровку: arccos1,8 — это угол, косинус которого равен 1,8… Скока-скока!? 1,8!? Косинус не бывает больше единицы!!!

Верно. Выражение arccos1,8 не имеет смысла. И запись такого выражения в какой-нибудь ответ изрядно повеселит проверяющего.)

Элементарно, как видите.) У каждого угла имеется свой персональный синус и косинус. И почти у каждого — свой тангенс и котангенс. Стало быть, зная тригонометрическую функцию, можно записать и сам угол. Для этого и предназначены арксинусы, арккосинусы, арктангенсы и арккотангенсы. Далее я всю эту семейку буду называть уменьшительно — арки. Чтобы печатать меньше.)

Внимание! Элементарная словесная и осознанная расшифровка арков позволяет спокойно и уверенно решать самые различные задания. А в непривычных заданиях только она и спасает.

А можно переходить от арков к обычным градусам или радианам? — слышу осторожный вопрос.)

Почему — нет!? Легко. И туда можно, и обратно. Более того, это иногда нужно обязательно делать. Арки — штука простая, но без них как-то спокойнее, правда?)

Например: что такое arcsin 0,5?

Вспоминаем расшифровку: arcsin 0,5 — это угол, синус которого равен 0,5. Теперь включаем голову (или гугл)) и вспоминаем, у какого угла синус равен 0,5? Синус равен 0,5 у угла в 30 градусов . Вот и все дела: arcsin 0,5 — это угол 30°. Можно смело записать:

arcsin 0,5 = 30°

Или, более солидно, через радианы:

Всё, можно забыть про арксинус и работать дальше с привычными градусами или радианами.

Если вы осознали, что такое арксинус, арккосинус… Что такое арктангенс, арккотангенс… То легко разберётесь, например, с таким монстром.)

Несведущий человек отшатнётся в ужасе, да…) А сведущий вспомнит расшифровку: арксинус — это угол, синус которого… Ну и так далее. Если сведущий человек знает ещё и таблицу синусов… Таблицу косинусов. Таблицу тангенсов и котангенсов, то проблем вообще нет!

Достаточно сообразить, что:

Расшифрую, т.е. переведу формулу в слова: угол, тангенс которого равен 1 (arctg1) — это угол 45°. Или, что едино, Пи/4. Аналогично:

и всё… Заменяем все арки на значения в радианах, всё посокращается, останется посчитать, сколько будет 1+1. Это будет 2.) Что и является правильным ответом.

Вот таким образом можно (и нужно) переходить от арксинусов, арккосинусов, арктангенсов и арккотангенсов к обычным градусам и радианам. Это здорово упрощает страшные примеры!

Частенько, в подобных примерах, внутри арков стоят отрицательные значения. Типа, arctg(-1,3), или, к примеру, arccos(-0,8)… Это не проблема. Вот вам простые формулы перехода от отрицательных значений к положительным:

Нужно вам, скажем, определить значение выражения:

Это можно и по тригонометрическому кругу решить, но вам не хочется его рисовать. Ну и ладно. Переходим от отрицательного значения внутри арккосинуса к положительному по второй формуле:

Внутри арккосинуса справа уже положительное значение. То, что

вы просто обязаны знать. Остаётся подставить радианы вместо арккосинуса и посчитать ответ:

Вот и всё.

Ограничения на арксинус, арккосинус, арктангенс, арккотангенс.

С примерами 7 — 9 проблема? Ну да, есть там некоторая хитрость. )

Все эти примеры, с 1-го по 9-й, тщательно разобраны по полочкам в Разделе 555. Что, как и почему. Со всеми тайными ловушками и подвохами. Плюс способы резкого упрощения решения. Кстати, в этом разделе много полезной информации и практических советов по тригонометрии в целом. И не только по тригонометрии. Очень помогает.

Если Вам нравится этот сайт…

Кстати, у меня есть ещё парочка интересных сайтов для Вас.)

Можно потренироваться в решении примеров и узнать свой уровень. Тестирование с мгновенной проверкой. Учимся — с интересом!)

можно познакомиться с функциями и производными.

Если говорить просто, то это овощи, приготовленные в воде по специальному рецепту. Я буду рассматривать два исходных компонента (овощной салат и воду) и готовый результат — борщ. Геометрически это можно представить как прямоугольник, в котором одна сторона обозначает салат, вторая сторона обозначает воду. Сумма этих двух сторон будет обозначать борщ. Диагональ и площадь такого «борщевого» прямоугольника являются чисто математическими понятиями и никогда не используются в рецептах приготовления борща.


Как салат и вода превращаются в борщ с точки зрения математики? Как сумма двух отрезков может превратиться в тригонометрию? Чтобы понять это, нам понадобятся линейные угловые функции.


В учебниках математики вы ничего не найдете о линейных угловых функциях. А ведь без них не может быть математики. Законы математики, как и законы природы, работают независимо от того, знаем мы о их существовании или нет.

Линейные угловые функции — это законы сложения. Посмотрите, как алгебра превращается в геометрию, а геометрия превращается в тригонометрию.

Можно ли обойтись без линейных угловых функций? Можно, ведь математики до сих пор без них обходятся. Хитрость математиков заключается в том, что они всегда рассказывают нам только о тех задачах, которые они сами умеют решать, и никогда не рассказывают о тех задачах, которые они решать не умеют. Смотрите. Если нам известен результат сложения и одно слагаемое, для поиска другого слагаемого мы используем вычитание. Всё. Других задач мы не знаем и решать не умеем. Что делать в том случае, если нам известен только результат сложения и не известны оба слагаемые? В этом случае результат сложения нужно разложить на два слагаемых при помощи линейных угловых функций. Дальше мы уже сами выбираем, каким может быть одно слагаемое, а линейные угловые функции показывают, каким должно быть второе слагаемое, чтобы результат сложения был именно таким, какой нам нужен. Таких пар слагаемых может быть бесконечное множество. В повседневной жизни мы прекрасно обходимся без разложения суммы, нам достаточно вычитания. А вот при научных исследованиях законов природы разложение суммы на слагаемые очень может пригодиться.

Ещё один закон сложения, о котором математики не любят говорить (ещё одна их хитрость), требует, чтобы слагаемые имели одинаковые единицы измерения. Для салата, воды и борща это могут быть единицы измерения веса, объема, стоимости или единицы измерения.

На рисунке показаны два уровня различий для математических . Первый уровень — это различия в области чисел, которые обозначены a , b , c . Это то, чем занимаются математики. Второй уровень — это различия в области единиц измерения, которые показаны в квадратных скобках и обозначены буквой U . Этим занимаются физики. Мы же можем понимать третий уровень — различия в области описываемых объектов. Разные объекты могут иметь одинаковое количество одинаковых единиц измерения. Насколько это важно, мы можем увидеть на примере тригонометрии борща. Если мы добавим нижние индексы к одинаковому обозначению единиц измерения разных объектов, мы сможем точно говорить, какая математическая величина описывает конкретный объект и как она изменяется с течением времени или в связи с нашими действиями. Буквой W я обозначу воду, буквой S обозначу салат и буквой B — борщ. Вот как будут выглядеть линейные угловые функции для борща.

Если мы возьмем какую-то часть воды и какую-то часть салата, вместе они превратятся в одну порцию борща. Здесь я предлагаю вам немного отвлечься от борща и вспомнить далекое детство. Помните, как нас учили складывать вместе зайчиков и уточек? Нужно было найти, сколько всего зверушек получится. Что же нас тогда учили делать? Нас учили отрывать единицы измерения от чисел и складывать числа. Да, одно любое число можно сложить с другим любым числом. Это прямой путь к аутизму современной математики — мы делаем непонятно что, непонятно зачем и очень плохо понимаем, как это относится к реальности, ведь из трех уровней различия математики оперируют только одним. Более правильно будет научиться переходить от одних единиц измерения к другим.

И зайчиков, и уточек, и зверушек можно посчитать в штуках. Одна общая единица измерения для разных объектов позволяет нам сложить их вместе. Это детский вариант задачи. Давайте посмотрим на похожую задачу для взрослых. Что получится, если сложить зайчиков и деньги? Здесь можно предложить два варианта решения.

Первый вариант . Определяем рыночную стоимость зайчиков и складываем её с имеющейся денежной суммой. Мы получили общую стоимость нашего богатства в денежном эквиваленте.

Второй вариант . Можно количество зайчиков сложить с количеством имеющихся у нас денежных купюр. Мы получим количество движимого имущества в штуках.

Как видите, один и тот же закон сложения позволяет получить разные результаты. Всё зависит от того, что именно мы хотим знать.

Но вернемся к нашему борщу. Теперь мы можем посмотреть, что будет происходить при разных значениях угла линейных угловых функций.

Угол равен нулю. У нас есть салат, но нет воды. Мы не можем приготовить борщ. Количество борща также равно нулю. Это совсем не значит, что ноль борща равен нулю воды. Ноль борща может быть и при нуле салата (прямой угол).


Лично для меня, это основное математическое доказательство того факта, что . Ноль не изменяет число при сложении. Это происходит потому, что само сложение невозможно, если есть только одно слагаемое и отсутствует второе слагаемое. Вы к этому можете относиться как угодно, но помните — все математические операции с нулем придумали сами математики, поэтому отбрасывайте свою логику и тупо зубрите определения, придуманные математиками: «деление на ноль невозможно», «любое число, умноженное на ноль, равняется нулю», «за выколом точки ноль» и прочий бред. Достаточно один раз запомнить, что ноль не является числом, и у вас уже никогда не возникнет вопрос, является ноль натуральным числом или нет, потому что такой вопрос вообще лишается всякого смысла: как можно считать числом то, что числом не является. Это всё равно, что спрашивать, к какому цвету отнести невидимый цвет. Прибавлять ноль к числу — это то же самое, что красить краской, которой нет. Сухой кисточкой помахали и говорим всем, что » мы покрасили». Но я немного отвлекся.

Угол больше нуля, но меньше сорока пяти градусов. У нас много салата, но мало воды. В результате мы получим густой борщ.

Угол равен сорок пять градусов. Мы имеем в равных количествах воду и салат. Это идеальный борщ (да простят меня повара, это просто математика).

Угол больше сорока пяти градусов, но меньше девяноста градусов. У нас много воды и мало салата. Получится жидкий борщ.

Прямой угол. У нас есть вода. От салата остались только воспоминания, поскольку угол мы продолжаем измерять от линии, которая когда-то обозначала салат. Мы не можем приготовить борщ. Количество борща равно нулю. В таком случае, держитесь и пейте воду, пока она есть)))

Вот. Как-то так. Я могу здесь рассказать и другие истории, которые будут здесь более чем уместны.

Два друга имели свои доли в общем бизнесе. После убийства одного из них, всё досталось другому.

Появление математики на нашей планете.

Все эти истории на языке математики рассказаны при помощи линейных угловых функций. Как-нибудь в другой раз я покажу вам реальное место этих функций в структуре математики. А пока, вернемся к тригонометрии борща и рассмотрим проекции.

суббота, 26 октября 2019 г.

среда, 7 августа 2019 г.

Завершая разговор о , нужно рассмотреть бесконечное множество. Дало в том, что понятие «бесконечность» действует на математиков, как удав на кролика. Трепетный ужас перед бесконечностью лишает математиков здравого смысла. Вот пример:

Первоисточник находится . Альфа обозначает действительное число. Знак равенства в приведенных выражениях свидетельствует о том, что если к бесконечности прибавить число или бесконечность, ничего не изменится, в результате получится такая же бесконечность. Если в качестве примера взять бесконечное множество натуральных чисел, то рассмотренные примеры можно представить в таком виде:

Для наглядного доказательства своей правоты математики придумали много разных методов . Лично я смотрю на все эти методы, как на пляски шаманов с бубнами. По существу, все они сводятся к тому, что либо часть номеров не занята и в них заселяются новые гости, либо к тому, что часть посетителей вышвыривают в коридор, чтобы освободить место для гостей (очень даже по-человечески). Свой взгляд на подобные решения я изложил в форме фантастического рассказа о Блондинке. На чем основываются мои рассуждения? Переселение бесконечного количества посетителей требует бесконечно много времени. После того, как мы освободили первую комнату для гостя, один из посетителей всегда будет идти по коридору из своего номера в соседний до скончания века. Конечно, фактор времени можно тупо игнорировать, но это уже будет из разряда «дуракам закон не писан». Всё зависит от того, чем мы занимаемся: подгоняем реальность под математические теории или наоборот.

Что же такое «бесконечная гостиница»? Бесконечная гостиница — это гостиница, в которой всегда есть любое количество свободных мест, независимо от того, сколько номеров занято. Если все номера в бесконечном коридоре «для посетителей» заняты, есть другой бесконечный коридор с номерами «для гостей». Таких коридоров будет бесконечное множество. При этом у «бесконечной гостиницы» бесконечное количество этажей в бесконечном количестве корпусов на бесконечном количестве планет в бесконечном количестве вселенных, созданных бесконечным количеством Богов. Математики же не способны отстраниться от банальных бытовых проблем: Бог-Аллах-Будда — всегда только один, гостиница — она одна, коридор — только один. Вот математики и пытаются подтасовывать порядковые номера гостиничных номеров, убеждая нас в том, что можно «впихнуть невпихуемое».

Логику своих рассуждений я вам продемонстрирую на примере бесконечного множества натуральных чисел. Для начала нужно ответить на очень простой вопрос: сколько множеств натуральных чисел существует — одно или много? Правильного ответа на это вопрос не существует, поскольку числа придумали мы сами, в Природе чисел не существует. Да, Природа отлично умеет считать, но для этого она использует другие математические инструменты, не привычные для нас. Как Природа считает, я вам расскажу в другой раз. Поскольку числа придумали мы, то мы сами будем решать, сколько множеств натуральных чисел существует. Рассмотрим оба варианта, как и подобает настоящим ученым.

Вариант первый. «Пусть нам дано» одно-единственное множество натуральных чисел, которое безмятежно лежит на полочке. Берем с полочки это множество. Всё, других натуральных чисел на полочке не осталось и взять их негде. Мы не можем к этому множеству прибавить единицу, поскольку она у нас уже есть. А если очень хочется? Без проблем. Мы можем взять единицу из уже взятого нами множества и вернуть её на полочку. После этого мы можем взять с полочки единицу и прибавить её к тому, что у нас осталось. В результате мы снова получим бесконечное множество натуральных чисел. Записать все наши манипуляции можно так:

Я записал действия в алгебраической системе обозначений и в системе обозначений, принятой в теории множеств, с детальным перечислением элементов множества. Нижний индекс указывает на то, что множество натуральных чисел у нас одно и единственное. Получается, что множество натуральных чисел останется неизменным только в том случае, если из него вычесть единицу и прибавить эту же единицу.

Вариант второй. У нас на полочке лежит много разных бесконечных множеств натуральных чисел. Подчеркиваю — РАЗНЫХ, не смотря на то, что они практически не отличимы. Берем одно из этих множеств. Потом из другого множества натуральных чисел берем единицу и прибавляем к уже взятому нами множеству. Мы можем даже сложить два множества натуральных чисел. Вот что у нас получится:

Нижние индексы «один» и «два» указывают на то, что эти элементы принадлежали разным множествам. Да, если к бесконечному множеству прибавить единицу, в результате получится тоже бесконечное множество, но оно не будет таким же, как первоначальное множество. Если к одному бесконечному множеству прибавить другое бесконечное множество, в результате получится новое бесконечное множество, состоящее из элементов первых двух множеств.

Множество натуральных чисел используется для счета так же, как линейка для измерений. Теперь представьте, что к линейке вы добавили один сантиметр. Это уже будет другая линейка, не равная первоначальной.

Вы можете принимать или не принимать мои рассуждения — это ваше личное дело. Но если когда-то вы столкнетесь с математическими проблемами, задумайтесь, не идете ли вы по тропе ложных рассуждений, протоптанной поколениями математиков. Ведь занятия математикой, прежде всего, формируют у нас устойчивый стереотип мышления, а уже потом добавляют нам умственных способностей (или наоборот, лишают нас свободомыслия).

pozg.ru

воскресенье, 4 августа 2019 г.

Дописывал постскриптум к статье о и увидел в Википедии этот замечательный текст:

Читаем: «… богатая теоретическая основа математики Вавилона не имела целостного характера и сводилась к набору разрозненных приемов, лишенных общей системы и доказательной базы.»

Вау! Какие мы умные и как хорошо можем видеть недостатки других. А слабо нам посмотреть на современную математику в таком же разрезе? Слегка перефразируя приведенный текст, лично у меня получилось следующее:

Богатая теоретическая основа современной математики не имеет целостного характера и сводится к набору разрозненных разделов, лишенных общей системы и доказательной базы.

За подтверждением своих слов я далеко ходить не буду — имеет язык и условные обозначения, отличные от языка и условных обозначений многих других разделов математики. Одни и те же названия в разных разделах математики могут иметь разный смысл. Наиболее очевидным ляпам современной математики я хочу посвятить целый цикл публикаций. До скорой встречи.

суббота, 3 августа 2019 г.

Как разделить множество на подмножества? Для этого необходимо ввести новую единицу измерения, присутствующую у части элементов выбранного множества. Рассмотрим пример.

Пусть у нас есть множество А , состоящее из четырех человек. Сформировано это множество по признаку «люди» Обозначим элементы этого множества через букву а , нижний индекс с цифрой будет указывать на порядковый номер каждого человека в этом множестве. Введем новую единицу измерения «половой признак» и обозначим её буквой b . Поскольку половые признаки присущи всем людям, умножаем каждый элемент множества А на половой признак b . Обратите внимание, что теперь наше множество «люди» превратилось в множество «люди с половыми признаками». После этого мы можем разделить половые признаки на мужские bm и женские bw половые признаки. Вот теперь мы можем применить математический фильтр: выбираем один из этих половых признаков, безразлично какой — мужской или женский. Если он присутствует у человека, тогда умножаем его на единицу, если такого признака нет — умножаем его на ноль. А дальше применяем обычную школьную математику. Смотрите, что получилось.

После умножения, сокращений и перегруппировок, мы получили два подмножества: подмножество мужчин Bm и подмножество женщин Bw . Приблизительно так же рассуждают математики, когда применяют теорию множеств на практике. Но в детали они нас не посвящают, а выдают готовый результат — «множество людей состоит из подмножества мужчин и подмножества женщин». Естественно, у вас может возникнуть вопрос, насколько правильно применена математика в изложенных выше преобразованиях? Смею вас заверить, по сути преобразований сделано всё правильно, достаточно знать математическое обоснование арифметики, булевой алгебры и других разделов математики. Что это такое? Как-нибудь в другой раз я вам об этом расскажу.

Что касается надмножеств, то объединить два множества в одно надмножество можно, подобрав единицу измерения, присутствующую у элементов этих двух множеств.

Как видите, единицы измерения и обычная математика превращают теорию множеств в пережиток прошлого. Признаком того, что с теорией множеств не всё в порядке, является то, что для теории множеств математики придумали собственный язык и собственные обозначения. Математики поступили так, как когда-то поступали шаманы. Только шаманы знают, как «правильно» применять их «знания». Этим «знаниям» они обучают нас.

В заключение, я хочу показать вам, как математики манипулируют с .

понедельник, 7 января 2019 г.

В пятом веке до нашей эры древнегреческий философ Зенон Элейский сформулировал свои знаменитые апории, самой известной из которых является апория «Ахиллес и черепаха». Вот как она звучит:

Допустим, Ахиллес бежит в десять раз быстрее, чем черепаха, и находится позади неё на расстоянии в тысячу шагов. За то время, за которое Ахиллес пробежит это расстояние, черепаха в ту же сторону проползёт сто шагов. Когда Ахиллес пробежит сто шагов, черепаха проползёт ещё десять шагов, и так далее. Процесс будет продолжаться до бесконечности, Ахиллес так никогда и не догонит черепаху.

Это рассуждение стало логическим шоком для всех последующих поколений. Аристотель, Диоген, Кант, Гегель, Гильберт… Все они так или иначе рассматривали апории Зенона. Шок оказался настолько сильным, что «… дискуссии продолжаются и в настоящее время, прийти к общему мнению о сущности парадоксов научному сообществу пока не удалось… к исследованию вопроса привлекались математический анализ, теория множеств, новые физические и философские подходы; ни один из них не стал общепризнанным решением вопроса… » [Википедия, » Апории Зенона «]. Все понимают, что их дурят, но никто не понимает, в чем заключается обман.

С точки зрения математики, Зенон в своей апории наглядно продемонстрировал переход от величины к . Этот переход подразумевает применение вместо постоянных. Насколько я понимаю, математический аппарат применения переменных единиц измерения либо ещё не разработан, либо его не применяли к апории Зенона. Применение же нашей обычной логики приводит нас в ловушку. Мы, по инерции мышления, применяем постоянные единицы измерения времени к обратной величине. С физической точки зрения это выглядит, как замедление времени до его полной остановки в момент, когда Ахиллес поравняется с черепахой. Если время останавливается, Ахиллес уже не может перегнать черепаху.

Если перевернуть привычную нам логику, всё становится на свои места. Ахиллес бежит с постоянной скоростью. Каждый последующий отрезок его пути в десять раз короче предыдущего. Соответственно, и время, затрачиваемое на его преодоление, в десять раз меньше предыдущего. Если применять понятие «бесконечность» в этой ситуации, то правильно будет говорить «Ахиллес бесконечно быстро догонит черепаху».

Как избежать этой логической ловушки? Оставаться в постоянных единицах измерения времени и не переходить к обратным величинам. На языке Зенона это выглядит так:

За то время, за которое Ахиллес пробежит тысячу шагов, черепаха в ту же сторону проползёт сто шагов. За следующий интервал времени, равный первому, Ахиллес пробежит ещё тысячу шагов, а черепаха проползет сто шагов. Теперь Ахиллес на восемьсот шагов опережает черепаху.

Этот подход адекватно описывает реальность без всяких логических парадоксов. Но это не полное решение проблемы. На Зеноновскую апорию «Ахиллес и черепаха» очень похоже утверждение Эйнштейна о непреодолимости скорости света. Эту проблему нам ещё предстоит изучить, переосмыслить и решить. И решение нужно искать не в бесконечно больших числах, а в единицах измерения.

Другая интересная апория Зенона повествует о летящей стреле:

Летящая стрела неподвижна, так как в каждый момент времени она покоится, а поскольку она покоится в каждый момент времени, то она покоится всегда.

В этой апории логический парадокс преодолевается очень просто — достаточно уточнить, что в каждый момент времени летящая стрела покоится в разных точках пространства, что, собственно, и является движением. Здесь нужно отметить другой момент. По одной фотографии автомобиля на дороге невозможно определить ни факт его движения, ни расстояние до него. Для определения факта движения автомобиля нужны две фотографии, сделанные из одной точки в разные моменты времени, но по ним нельзя определить расстояние. Для определения расстояния до автомобиля нужны две фотографии, сделанные из разных точек пространства в один момент времени, но по ним нельзя определить факт движения (естественно, ещё нужны дополнительные данные для расчетов, тригонометрия вам в помощь). На что я хочу обратить особое внимание, так это на то, что две точки во времени и две точки в пространстве — это разные вещи, которые не стоит путать, ведь они предоставляют разные возможности для исследования.
Покажу процесс на примере. Отбираем «красное твердое в пупырышку» — это наше «целое». При этом мы видим, что эти штучки есть с бантиком, а есть без бантика. После этого мы отбираем часть «целого» и формируем множество «с бантиком». Вот так шаманы добывают себе корм, привязывая свою теорию множеств к реальности.

А теперь сделаем маленькую пакость. Возьмем «твердое в пупырышку с бантиком» и объединим эти «целые» по цветовому признаку, отобрав красные элементы. Мы получили множество «красное». Теперь вопрос на засыпку: полученные множества «с бантиком» и «красное» — это одно и то же множество или два разных множества? Ответ знают только шаманы. Точнее, сами они ничего не знают, но как скажут, так и будет.

Этот простой пример показывает, что теория множеств совершенно бесполезна, когда речь заходит о реальности. В чем секрет? Мы сформировали множество «красное твердое в пупырышку с бантиком». Формирование происходило по четырем разным единицам измерения: цвет (красное), прочность (твердое), шероховатость (в пупырышку), украшения (с бантиком). Только совокупность единиц измерения позволяет адекватно описывать реальные объекты на языке математики . Вот как это выглядит.

Буква «а» с разными индексами обозначает разные единицы измерения. В скобках выделены единицы измерения, по которым выделяется «целое» на предварительном этапе. За скобки вынесена единица измерения, по которой формируется множество. Последняя строчка показывает окончательный результат — элемент множества. Как видите, если применять единицы измерения для формирования множества, тогда результат не зависит от порядка наших действий. А это уже математика, а не пляски шаманов с бубнами. Шаманы могут «интуитивно» придти к такому же результату, аргументируя его «очевидностью», ведь единицы измерения не входят в их «научный» арсенал.

При помощи единиц измерения очень легко разбить одно или объединить несколько множеств в одно надмножество. Давайте более внимательно рассмотрим алгебру этого процесса.

Функции и Графики — сайт по математике и не только!!! Всё о Математических функциях и их графиках…

ОБРАТНЫЕ ТРИГОНОМЕТРИЧЕСКИЕ ФУНКЦИИ

y = arcsin x

y = arccos x

функция обратная функции y = sin x,
— / 2 x / 2
функция обратная функции y = cos x,
0 x

Свойства функций

y = arcsin xy = arccos x
ОБЛАСТЬ ОПРЕДЕЛЕНИЯ:[-1; 1][-1; 1]
ОБЛАСТЬ ЗНАЧЕНИЙ:[0; )
ЧЕТНОСТЬ, НЕЧЕТНОСТЬ:нечетнаяни четная, ни нечетная
НУЛИ:y = 0 при x = 0 y = 0 при x = 1
ПРОМЕЖУТКИ ЗНАКОПОСТОЯНСТВА: y > 0, при x (0; ] y x [-1; 0) y = 0 при x = 1 y > 0 при x [-1; 1)
ЭКСТРЕМУМЫ:нетнет
ПРОМЕЖУТКИ МОНОТОННОСТИ:возрастает на всей области определенияубывает на всей области определения

arcsin x + arccos x = /2

y = arctg x

y = arcctg x

функция обратная функции y = tg x, — / 2 x / 2функция обратная функции y = ctg x, 0 x
y = arctg xy = arcctg x
ОБЛАСТЬ ОПРЕДЕЛЕНИЯ:RR
ОБЛАСТЬ ЗНАЧЕНИЙ:(0; )
ЧЕТНОСТЬ, НЕЧЕТНОСТЬ:нечетнаяни четная, ни нечетная
НУЛИ:y = 0 при x = 0 нулей нет
ПРОМЕЖУТКИ
ЗНАКОПОСТОЯНСТВА:
y > 0, при x (0; ] y x (-; 0)y > 0 при x R
ЭКСТРЕМУМЫ:нетнет
ПРОМЕЖУТКИ
МОНОТОННОСТИ:
возрастает при x Rубывает при x R

arctg x + arcctg x = /2

Пособие по теме Арксинус, арккосинус, арктангенс числа

ГОСУДАРСТВЕННОЕ АВТОНОМНОЕ ПРОФЕССИОНАЛЬНОЕ ОБРАЗОВАТЕЛЬНОЕ УЧРЕЖДЕНИЕ НОВОСИБИРСКОЙ ОБЛАСТИ «КУПИНСКИЙ МЕДИЦИНСКИЙ ТЕХНИКУМ»

МЕТОДИЧЕСКОЕ ПОСОБИЕ

Для самостоятельной работы студентов

По дисциплине: МАТЕМАТИКА: алгебра и начало математического анализа; геометрия

Тема: «АРКСИНУС, АРККОСИНУС, АРКТАНГЕНС ЧИСЛА»

Специальность: 34. 02.01 Сестринское дело Курс: 1

(базовой подготовки)

Купино

2019

Рассмотрено на заседании предметной цикловой

Методической комиссии по общеобразовательным дисциплинам,

общему гуманитарному и социально-экономическому, математическому и

естественно-научному циклу

Протокол № _____ от «_____» _________20____г.

Автор – составитель: преподаватель математики высшей категории Тюменцева О.Н.

Купино

2019 г

Пояснительная записка к методическому пособию

Методическое пособие предназначено для повторения теоретических и практических знаний по теме.

Цель пособия – повторить понятия: тригонометрических функций, радианной меры углов, таблицы значений тригонометрических функций, формулы перевода градусов в радианы и наоборот, определения арксинуса, арккосинуса, арктангенса числа и подготовится к занятию по теме «Арксинус, арккосинус, арктангенс числа».

Данное пособие рекомендовано для студентов первого курса специальности 34.02.01 Сестринское дело. Пособие содержит определения, свойства и формулы по теме: Арксинус, арккосинус, арктангенс числа, тест для самоконтроля и ключи к тесту.

Пособие направлено на формирование навыков самостоятельной работы с учебным материалом, формирование навыков решения задач, формирование и развитие творческого потенциала, повышение интереса к дисциплине.

Арксинус, арккосинус, арктангенс числа

Арксинус

Чтобы больше понять о свойствах арксинуса, необходимо рассмотреть его функцию. График y = arcsin x имеет вид асимметричной кривой, проходящей через центр координат.

Свойства арксинуса:

  1.  

  2. Так как f(x) нечетная, то arcsin (- x) = — arcsin x.

  3. Y = 0 при x = 0.

  4. На всей своей протяженности график возрастает.

Если сопоставить графики sin и arcsin, у двух тригонометрических функций можно найти общие закономерности.

Арккосинус

Arccos числа а — это значение угла α, косинус которого равен а.

Кривая y = arcos x зеркально отображает график arcsin x, с той лишь разницей, что проходит через точку π/2 на оси OY.

Рассмотрим функцию арккосинуса более подробно:

  1. Функция определена на отрезке [-1; 1].

  2. ОДЗ для arccos — [0, π].

  3. График целиком расположен в I и II четвертях, а сама функция не является ни четной, ни нечетной.

  4. Y = 0 при x = 1.

  5. Кривая убывает на всей своей протяженности. Некоторые свойства арккосинуса совпадают с функцией косинуса.

Некоторые свойства арккосинуса совпадают с функцией косинуса.

Задание 1. Укажите функции изображенные на рисунке.

Ответ: рис. 1 – 4, рис.2 — 1.

Арктангенс

Arctg числа a – это такое значение угла α, что его тангенс равен а.

Если рассмотреть график арктангенса, можно выделить следующие свойства:

  1. График бесконечен и определен на промежутке (- ∞; + ∞).

  2. Арктангенс нечетная функция, следовательно, arctg (- x) = — arctg x.

  3. Y = 0 при x = 0.

  4. Кривая возрастает на всей области определения.

Приведем краткий сравнительный анализ tg x и arctg x в виде таблицы.

Арккотангенс

Arcctg числа a — принимает такое значение α из интервала (0; π), что его котангенс равен а.

Свойства функции арккотангенса:

  1. Интервал определения функции – бесконечность.

  2. Область допустимых значений – промежуток (0; π).

  3. F(x) не является ни четной, ни нечетной.

  4. На всем своем протяжении график функции убывает.

 Сопоставить ctg x и arctg x очень просто, нужно лишь сделать два рисунка и описать поведение кривых.

Задание 2. Соотнести график и форму записи функции.

Если рассуждать логически, из графиков видно, что обе функции возрастающие. Следовательно, оба рисунка отображают некую функцию arctg.  Из свойств арктангенса известно, что y=0 при x = 0,

Ответ: рис. 1 – 1, рис. 2 – 4.

Пример 1. Вычислить значение  .

Решение.  Если обозначить  , то  . Из определения функции  следует, что   и . Так как  , то    и .

Однако  ,  поэтому  .

Ответ:   .  

Пример 2. Вычислить значение  .

Решение.  Если  , то  . Согласно определению функции  ,  имеем    и  . Так как  , то   .  

Поскольку    и  , то    . 

Ответ:   .

Пример 3. Вычислить значение  .

Решение.  Пусть  ,  тогда   и , где  .  В таком случае    и  .

Поскольку   , то  . 

Ответ: . 

Пример 4. Вычислить значение  . Решение. Так как , то   или . Обозначим  , тогда  ,  . Поскольку  , то  .

Если  , то  ,   или  . Однако  , поэтому  ,   и  .

Ответ:  . 

Пример 5. Вычислить значение  . 

Решение.  Если положить   и , то  или  

                                         ,                                (1) 

где из определения функции  имеем  и .

Однако  и  , поэтому   и  . В таком случае  ,    и  ,  .

Если  ,  то  ,   или  .  Поскольку    , то .

С учетом того, что   и  , имеем     и  .

Если  , то повторяя рассуждения, приведенные выше, получаем  ,   и  , .

Подставляя значения , , ,  в выражение (1) получаем  .

Ответ:  .

Примечание.  Так как в примере 5 показано, что  , где  ,   и  , то справедливо равенство

                                         .                                         

Отсюда также следует, что  . 

Пример 6. Вычислить значение  . 

Решение.  Из определения функции    следует, что .  Из условия примера получаем  .

Так как  , то имеем уравнение   или   .  Далее, принимая во внимание теорему 1, записываем две серии корней уравнения    вида

  и   , 

где   целые числа.  Если положить  , то из второй серии корней вытекает единственное значение , которое удовлетворяет двойному неравенству  .

Ответ:  .

Пример 7. Вычислить значение  .

Решение.  По определению функции    имеем  . Из условия следует, что  ,    или

                                            .                                   (2)

Согласно теореме 2, здесь имеем две серии корней уравнения (2): 

  и   , где   целые числа.

Так как  , то из первой серии корней при условии, что  ,  получаем  .

Ответ:  .

Тест по теме Арксинус, арккосинус, арктангенс числа

Найдите значение выражения:
  1. аrcsin (-

а) ; б) — ; в)

2. arcos (-1)

а) ; б) — ; в) 0

  1. arctg

а) ; б) — ; в) 0

  1. arctg (- 1) + arcsin 0

а) — ; б) ; в) 0

  1. аrcsin (-) + arcsin 0

а) ; б) ; в) г) нет верного ответа

  1. sin (arcsin )

а) ; б) — ; в) 0

  1. sin (arcsin 0)

а) 0; б) ; в) 1

Вариант 2

Найдите значение выражения:

  1. аrcsin ( — )

а) ; б) ; в) —

  1. arcos (-)

а) -; б) ; в)

  1. arctg

а) ; б) — ; в) 1

  1. arctg 1 + arcos 1

а) ; б) 0; в)

  1. аrcsin (-) + arcos (-)

а) ; б) ; в) —

  1. cos (arcos

а) ; б) — ; в) 1

  1. arcos (cos )

а) ; б) ; в) —

Ключ к тесту по теме Арксинус, арккосинус, арктангенс числа

п/п

Вариант

1

2

3

4

5

6

7

1

б

а

а

а

г

а

а

2

в

б

а

а

а

а

б

Критерии оценивания тестовых заданий

7 вопросов 5 (отлично) (7 ответов)

7 вопросов 4 (хорошо) (6 ответов)

7 вопросов 3 (удов) (5 ответов)

Литература

  1. Алимов Ш. А. и др. Алгебра и начала анализа. 10 (11) кл. – М.: 2018

  2. Башмаков М.И. Сборник задач: учеб. пособие (базовый уровень). 11 кл. М.: 2012

Интернет-ресурсы

  1. http://school-collection.edu.ru – Электронный учебник «Математика в

школе, XXI век».

  1. http://fcior.edu.ru — информационные, тренировочные и контрольные материалы.

  2. www.school-collection.edu.ru – Единая коллекции Цифровых образовательных ресурсов

Свойства обратных тригонометрических функций ⋆ ДПА и ЗНО онлайн

Так как геометрически значение обратной тригонометрической функции связано с длиной дуги единичной окружности (или углом, стягивающим эту дугу), соответствующей тому или иному отрезку, то названия обратных тригонометрических функций образуются следующим образом: приставка «арк-» (от латинского arc — дуга) + соответствующие им названия  тригонометрических функций.

Арксинус

Арксинусом числа $$a$$ называется такое значение угла $$\alpha,$$ для которого $$\sin \alpha=a,\;|a|\leqslant 1,\;\alpha\in[-\frac{\pi}{2};\frac{\pi}{2}].$$

  • Областью определения функции арксинус является отрезок $$[-1;1].$$
  • Областью значений функции арксинус является отрезок $$[-\frac{\pi}{2};\frac{\pi}{2}].$$
  • Арксинус строго возрастающая функция.
  • $$\sin \left (\arcsin a \right )=a,\;|a|\leqslant 1.$$
  • $$\arcsin\left (\sin \alpha \right )=\alpha,\;\alpha\in[-\frac{\pi}{2};\frac{\pi}{2}].$$
  • Арксинус является нечетной функцией: $$\arcsin(-a)=-\arcsin a,\;|a| \leqslant 1.$$
  • $$\arcsin a>0,\;a\in(0;1].$$
  • $$\arcsin a=0,\;a=0.$$
  • $$\arcsin a<0,\;a\in[-1;0).$$

Арккосинус

Арккосинусом числа $$a$$ называется такое значение угла $$\alpha,$$ для которого $$\cos \alpha=a,\;|a|\leqslant 1,\;\alpha\in[0;\pi].$$

  • Областью определения функции арккосинус является отрезок $$[-1;1]. $$
  • Областью значений функции арккосинус является отрезок $$[0;\pi].$$
  • Арккосинус строго убывающая функция.
  • $$\cos \left (\arccos a \right )=a,\;|a|\leqslant 1.$$
  • $$\arccos\left (\cos \alpha\right )=\alpha,\;\alpha\in[0;\pi].$$
  • Арккосинус является индифферентной функцией: $$\arccos (-a)=\pi-\arccos a,\;|a|\leqslant 1.$$ Функция центрально-симметрична относительно точки $$\left ( 0;\frac{\pi}{2} \right ).$$
  • $$\arccos a>0,\;a\in[-1;1).$$
  • $$\arccos a=0,\;a=1.$$

Арктангенс

Арктангенсом числа $$a$$ называется такое значение угла $$\alpha,$$ для которого $$\text{tg}\, \alpha=a,\;a\in\mathbb{R},\;\alpha\in\left (-\frac{\pi}{2};\frac{\pi}{2} \right ).$$

  • Областью определения функции арктангенс является вся числовая прямая: $$\mathbb{R}.$$
  • Областью значений функции арктангенс является интервал $$\left (-\frac{\pi}{2};\frac{\pi}{2} \right ).$$
  • Арктангенс строго возрастающая функция.
  • $$\text{tg}\left (\text{arctg}\,a \right ) =a,\;a\in\mathbb{R}. $$
  • $$\text{arctg}\left (\text{tg}\,\alpha \right ) =\alpha,\;\alpha\in\left ( -\frac{\pi}{2};\frac{\pi}{2} \right ).$$
  • Арктангенс является нечетной функцией: $$\text{arctg}\left (-a \right ) =-\text{arctg}\,a,\;a\in\mathbb{R}.$$
  • $$\text{arctg}\,a>0,\;a\in(0;\infty ).$$
  • $$\text{arctg}\,a=0,\;a=0.$$
  • $$\text{arctg}\,a<0,\;a\in(-\infty;0).$$

Арккотангенс

Арккотангенсом числа $$a$$ называется такое значение угла $$\alpha,$$ для которого $$\text{ctg}\, \alpha=a,\;a\in\mathbb{R},\;\alpha\in\left (0;\pi \right ).$$

  • Областью определения функции арккотангенс является вся числовая прямая: $$\mathbb{R}.$$
  • Областью значений функции арккотангенс является интервал $$\left (0;\pi \right ).$$
  • Арккотангенс строго убывающая функция.
  • $$\text{ctg}\left (\text{arcctg}\,a \right ) =a,\;a\in\mathbb{R}.$$
  • $$\text{arcctg}\left (\text{ctg}\,\alpha \right ) =\alpha,\;\alpha\in\left (0;\pi \right ).$$
  • Арккотангенс является индифферентной функцией: $$\text{arcctg}\left (-a \right ) =\pi-\text{arcctg}\,a,\;a\in\mathbb{R}. $$ Функция центрально-симметрична относительно точки $$\left ( 0;\frac{\pi}{2} \right ).$$
  • $$\text{arcctg}\,a>0,\;a\in\mathbb{R}.$$

Основные соотношения

  • $$\arcsin a+\arccos a=\frac{\pi}{2},\;|a|\leqslant 1.$$
  • $$\text{arctg}\,a+\text{arcctg}\,a=\frac{\pi}{2},\;a\in\mathbb{R}.$$

График arccos. Тригонометрия. Обратные тригонометрические функции. Арксинус. Получение функции arcsin

Определение и обозначения

Арксинус (y = arcsin x ) — это функция, обратная к синусу (x = sin y -1 ≤ x ≤ 1 и множество значений -π/2 ≤ y ≤ π/2 .
sin(arcsin x) = x ;
arcsin(sin x) = x .

Арксинус иногда обозначают так:
.

График функции арксинус

График функции y = arcsin x

График арксинуса получается из графика синуса, если поменять местами оси абсцисс и ординат. Чтобы устранить многозначность, область значений ограничивают интервалом , на котором функция монотонна. Такое определение называют главным значением арксинуса.

Арккосинус, arccos

Определение и обозначения

Арккосинус (y = arccos x ) — это функция, обратная к косинусу (x = cos y ). Он имеет область определения -1 ≤ x ≤ 1 и множество значений 0 ≤ y ≤ π .
cos(arccos x) = x ;
arccos(cos x) = x .

Арккосинус иногда обозначают так:
.

График функции арккосинус


График функции y = arccos x

График арккосинуса получается из графика косинуса, если поменять местами оси абсцисс и ординат. Чтобы устранить многозначность, область значений ограничивают интервалом , на котором функция монотонна. Такое определение называют главным значением арккосинуса.

Четность

Функция арксинус является нечетной:
arcsin(- x) = arcsin(-sin arcsin x) = arcsin(sin(-arcsin x)) = — arcsin x

Функция арккосинус не является четной или нечетной:
arccos(- x) = arccos(-cos arccos x) = arccos(cos(π-arccos x)) = π — arccos x ≠ ± arccos x

Свойства — экстремумы, возрастание, убывание

Функции арксинус и арккосинус непрерывны на своей области определения (см. доказательство непрерывности). Основные свойства арксинуса и арккосинуса представлены в таблице.

y = arcsin x y = arccos x
Область определения и непрерывность — 1 ≤ x ≤ 1 — 1 ≤ x ≤ 1
Область значений
Возрастание, убывание монотонно возрастает монотонно убывает
Максимумы
Минимумы
Нули, y = 0 x = 0 x = 1
Точки пересечения с осью ординат, x = 0 y = 0 y = π/2

Таблица арксинусов и арккосинусов

В данной таблице представлены значения арксинусов и арккосинусов, в градусах и радианах, при некоторых значениях аргумента.

x arcsin x arccos x
град. рад. град. рад.
— 1 — 90° 180° π
— 60° 150°
— 45° 135°
— 30° 120°
0 0 90°
30° 60°
45° 45°
60° 30°
1 90° 0

≈ 0,7071067811865476
≈ 0,8660254037844386

Формулы

См. также: Вывод формул обратных тригонометрических функций

Формулы суммы и разности


при или

при и

при и


при или

при и

при и


при

при


при

при

Выражения через логарифм, комплексные числа

См. также: Вывод формул

Выражения через гиперболические функции

Производные

;
.
См. Вывод производных арксинуса и арккосинуса > > >

Производные высших порядков :
,
где — многочлен степени . Он определяется по формулам:
;
;
.

См. Вывод производных высших порядков арксинуса и арккосинуса > > >

Интегралы

Делаем подстановку x = sin t . Интегрируем по частям, учитывая что -π/2 ≤ t ≤ π/2 , cos t ≥ 0 :
.

Выразим арккосинус через арксинус:
.

Разложение в ряд

При |x| 1 имеет место следующее разложение:
;
.

Обратные функции

Обратными к арксинусу и арккосинусу являются синус и косинус , соответственно.

Следующие формулы справедливы на всей области определения:
sin(arcsin x) = x
cos(arccos x) = x .

Следующие формулы справедливы только на множестве значений арксинуса и арккосинуса:
arcsin(sin x) = x при
arccos(cos x) = x при .

Использованная литература:
И.Н. Бронштейн, К.А. Семендяев, Справочник по математике для инженеров и учащихся втузов, «Лань», 2009.

См. также:

(круговые функции, аркфункции) — математические функции, которые являются обратными к тригонометрическим функциям .

Арккосинус , обратная функция к cos (x = cos y), y = arccos x определен при и имеет множество значений . Другими словами возвращает угол по значению его cos .

Арккосинус (обозначение: arccos x ; arccos x — это угол , косинус которого равняется x и так далее).

Функция y = cos x непрерывна и ограничена на всей своей числовой прямой. Функция y = arccos x является строго убывающей.

Свойства функции arcsin .

Получение функции arccos .

Дана функция y = cos x . На всей своей области определения она является кусочно-монотонной, и, значит, обратное соответствие y = arccos x функцией не является. Поэтому мы рассмотрим отрезок, на котором она строго убывает и принимает все свои значения — . На этом отрезке y = cos x строго монотонно убывает и принимает все свои значения только один раз, а значит, на отрезке существует обратная функция y = arccos x , график которой симметричен графику y = cos x на отрезке относительно прямой y = x .

Поскольку тригонометрические функции периодичны, то обратные к ним функции не однозначны. Так, уравнение y = sin x , при заданном , имеет бесконечно много корней. Действительно, в силу периодичности синуса, если x такой корень, то и x + 2πn (где n целое) тоже будет корнем уравнения. Таким образом, обратные тригонометрические функции многозначны . Чтобы с ними было проще работать, вводят понятие их главных значений. Рассмотрим, например, синус: y = sin x . Если ограничить аргумент x интервалом , то на нем функция y = sin x монотонно возрастает. Поэтому она имеет однозначную обратную функцию, которую называют арксинусом: x = arcsin y .

Если особо не оговорено, то под обратными тригонометрическими функциями имеют в виду их главные значения, которые определяются следующими определениями.

Арксинус (y = arcsin x ) — это функция, обратная к синусу (x = sin y
Арккосинус (y = arccos x ) — это функция, обратная к косинусу (x = cos y ), имеющая область определения и множество значений .
Арктангенс (y = arctg x ) — это функция, обратная к тангенсу (x = tg y ), имеющая область определения и множество значений .
Арккотангенс (y = arcctg x ) — это функция, обратная к котангенсу (x = ctg y ), имеющая область определения и множество значений .

Графики обратных тригонометрических функций

Графики обратных тригонометрических функций получаются из графиков тригонометрических функций зеркальным отражением относительно прямой y = x . См. разделы Синус, косинус , Тангенс, котангенс .

y = arcsin x


y = arccos x


y = arctg x


y = arcctg x

Основные формулы

Здесь следует особо обратить внимание на интервалы, для которых справедливы формулы.

arcsin(sin x) = x при
sin(arcsin x) = x
arccos(cos x) = x при
cos(arccos x) = x

arctg(tg x) = x при
tg(arctg x) = x
arcctg(ctg x) = x при
ctg(arcctg x) = x

Формулы, связывающие обратные тригонометрические функции

См. также: Вывод формул обратных тригонометрических функций

Формулы суммы и разности


при или

при и

при и


при или

при и

при и


при

при


при

при


при

при

при


при

при

при

Использованная литература:
И.Н. Бронштейн, К.А. Семендяев, Справочник по математике для инженеров и учащихся втузов, «Лань», 2009.

Функции sin, cos, tg и ctg всегда сопровождаются арксинусом, арккосинусом, арктангенсом и арккотангенсом. Одно является следствием другого, а пары функций одинаково важны для работы с тригонометрическими выражениями.

Рассмотрим рисунок единичной окружности, на котором графически отображено значений тригонометрических функций.

Если вычислить arcs OA, arcos OC, arctg DE и arcctg MK, то все они будут равны значению угла α. Формулы, приведенные ниже, отражают взаимосвязь основных тригонометрических функций и соответствующих им арков.

Чтобы больше понять о свойствах арксинуса, необходимо рассмотреть его функцию. График имеет вид асимметричной кривой, проходящей через центр координат.

Свойства арксинуса:

Если сопоставить графики sin и arcsin , у двух тригонометрических функций можно найти общие закономерности.

Арккосинус

Arccos числа а — это значение угла α, косинус которого равен а.

Кривая y = arcos x зеркально отображает график arcsin x, с той лишь разницей, что проходит через точку π/2 на оси OY.

Рассмотрим функцию арккосинуса более подробно:

  1. Функция определена на отрезке [-1; 1].
  2. ОДЗ для arccos — .
  3. График целиком расположен в I и II четвертях, а сама функция не является ни четной, ни нечетной.
  4. Y = 0 при x = 1.
  5. Кривая убывает на всей своей протяженности. Некоторые свойства арккосинуса совпадают с функцией косинуса.

Некоторые свойства арккосинуса совпадают с функцией косинуса.

Возможно, школьникам покажется излишним такое «подробное» изучение «арков». Однако, в противном случае, некоторые элементарные типовые задания ЕГЭ могут ввести учащихся в тупик.

Задание 1. Укажите функции изображенные на рисунке.

Ответ: рис. 1 – 4, рис.2 — 1.

В данном примере упор сделан на мелочах. Обычно ученики очень невнимательно относятся к построению графиков и внешнему виду функций. Действительно, зачем запоминать вид кривой, если ее всегда можно построить по расчетным точкам. Не стоит забывать, что в условиях теста время, затраченное на рисунок для простого задания, потребуется для решения более сложных заданий.

Арктангенс

Arctg числа a – это такое значение угла α, что его тангенс равен а.

Если рассмотреть график арктангенса, можно выделить следующие свойства:

  1. График бесконечен и определен на промежутке (- ∞; + ∞).
  2. Арктангенс нечетная функция, следовательно, arctg (- x) = — arctg x.
  3. Y = 0 при x = 0.
  4. Кривая возрастает на всей области определения.

Приведем краткий сравнительный анализ tg x и arctg x в виде таблицы.

Арккотангенс

Arcctg числа a — принимает такое значение α из интервала (0; π), что его котангенс равен а.

Свойства функции арккотангенса:

  1. Интервал определения функции – бесконечность.
  2. Область допустимых значений – промежуток (0; π).
  3. F(x) не является ни четной, ни нечетной.
  4. На всем своем протяжении график функции убывает.

Сопоставить ctg x и arctg x очень просто, нужно лишь сделать два рисунка и описать поведение кривых.

Задание 2. Соотнести график и форму записи функции.

Если рассуждать логически, из графиков видно, что обе функции возрастающие. Следовательно, оба рисунка отображают некую функцию arctg. Из свойств арктангенса известно, что y=0 при x = 0,

Ответ: рис. 1 – 1, рис. 2 – 4.

Тригонометрические тождества arcsin, arcos, arctg и arcctg

Ранее нами уже была выявлена взаимосвязь между арками и основными функциями тригонометрии. Данная зависимость может быть выражена рядом формул, позволяющих выразить, например, синус аргумента, через его арксинус, арккосинус или наоборот. Знание подобных тождеств бывает полезным при решении конкретных примеров.

Также существуют соотношения для arctg и arcctg:

Еще одна полезная пара формул, устанавливает значение для суммы значений arcsin и arcos, а также arcctg и arcctg одного и того же угла.

Примеры решения задач

Задания по тригонометрии можно условно разделить на четыре группы: вычислить числовое значение конкретного выражения, построить график данной функции, найти ее область определения или ОДЗ и выполнить аналитические преображения для решения примера.

При решении первого типа задач необходимо придерживаться следующего плана действий:

При работе с графиками функций главное – это знание их свойств и внешнего вида кривой. Для решения тригонометрических уравнений и неравенств необходимы таблицы тождеств. Чем больше формул помнит школьник, тем проще найти ответ задания.

Допустим в ЕГЭ необходимо найти ответ для уравнения типа:

Если правильно преобразовать выражение и привести к нужному виду, то решить его очень просто и быстро. Для начала, перенесем arcsin x в правую часть равенства.

Если вспомнить формулу arcsin (sin α) = α , то можно свести поиск ответов к решению системы из двух уравнений:

Ограничение на модель x возникло, опять таки из свойств arcsin: ОДЗ для x [-1; 1]. При а ≠0, часть сиcтемы представляет собой квадратное уравнение с корнями x1 = 1 и x2 = — 1/a. При a = 0, x будет равен 1.

Обратные тригонометрические функции (круговые функции, аркфункции) — математические функции, которые являются обратными к тригонометрическим функциям .

Арксинус (обозначается как arcsin x ; arcsin x — это угол, sin его равняется x ).

Арксинус (y = arcsin x ) — обратная тригонометрическая функция к sin (x = sin y ), которая имеет область определения и множество значений . Другими словами возвращает угол по значению его sin .

Функция y=sin x непрерывна и ограничена на всей своей числовой прямой. Функция y=arcsin x — строго возрастает.

Свойства функции arcsin .

График арксинуса.

Получение функции arcsin .

Есть функция y = sin x . На всей своей области определения она кусочно-монотонная, таким образом, обратное соответствие y = arcsin x не является функцией. Поэтому рассматриваем отрезок, на котором она только возрастает и принимает каждое значение области значений — . Т.к. для функции y = sin x на интервале все значения функции получается при только одном значении аргумента, значит, на этом отрезке есть обратная функция y = arcsin x , у которой график является симметричным графику функции y = sin x на отрезке относительно прямой y = x . 3 6 Risolvere per ? cos(x)=1/2 7 Risolvere per x sin(x)=-1/2 8 Преобразовать из градусов в радианы 225 9 Risolvere per ? cos(x)=( квадратный корень 2)/2 10 Risolvere per x cos(x)=( квадратный корень 3)/2 11 Risolvere per x sin(x)=( квадратный корень 3)/2 12 График g(x)=3/4* корень пятой степени x 13 Найти центр и радиус x^2+y^2=9 14 Преобразовать из градусов в радианы 120 град. 2+n-72)=1/(n+9)

Программа элективного курса для учащихся 11-го класса «Обратные тригонометрические функции»

Пояснительная записка

Предлагаемый элективный курс для учащихся 11-го класса посвящен одному из важнейших понятий математики. Понятия арксинуса, арккосинуса, арктангенса и арккотангенса вводятся в курс алгебры и начал анализа во время изучения учащимися простейших тригонометрических уравнений. При этом следует заметить, что практически все старшеклассники плохо знают, а тем более понимают, эти определения. Что же тогда говорить об обратных тригонометрических функциях?

В последнее время в материалах ЕГЭ и вступительных экзаменов в высшие учебные заведения, часто предлагаются задания по данной теме. Такие задачи вызывают затруднения у учащихся, так как практических заданий по этой теме в школьных учебниках мало.

Цель данного элективного курса – повысить математическую культуру учащихся в рамках школьной программы по математике, прояснить и дополнить школьный материал, связанный с обратными тригонометрическими функциями, представить его систематизацию и помочь старшеклассникам успешно сдать ЕГЭ по математике.

В курсе заложена возможность дифференцированного обучения, как путем использования задач различного уровня сложности, так и на основе различной степени самостоятельности осваивания нового материала. Следовательно, программа применима для самых различных групп школьников, в том числе не имеющих хорошей подготовки.

На изучение всего курса отводится 11 часов, по окончании предусмотрено зачетное мероприятие на 2 часа, а также возможны и другие формы комбинированной диагностики.

Учебно-тематический план

№ п/п

Тема

Количество часов

Форма контроля

1.

Определения арксинуса, арккосинуса, арктангенса и арккотангенса

1

Математический диктант

2.

Функции у=arcsin x, y=arccos x их графики и свойства.

1

Работа с таблицами с последующей взаимопроверкой

3.

Функции у=arcsin x, y=arccos x их графики и свойства.

1

Самостоятельная работа обучающего характера

4.

Функции у=arcsin x, y=arccos x, их графики и свойства.

1

Тест (различные уровни сложности)

5.

Функции у=arctg x, y=arcctg x, их графики и свойства.

1

Самостоятельная работа обучающего характера

6.

Функции у=arctg x, y=arcctg x, их графики и свойства.

1

Урок взаимопроверки

7.

Функции у=arctg x, y=arcctg x, их графики и свойства.

1

Тест (различные уровни сложности)

8.

Обобщающий урок по теме: “Обратные тригонометрические функции, их графики и свойства”

2

Практикум, работа в группах. Домашняя контрольная работа.

9.

Итоговый контроль

2

Зачет (тест)

Содержание

Тема 1. Определения арксинуса, арккосинуса, арктангенса и арккотангенса.

На первом занятии учащимся сообщается цель и значение данного курса. Определения арксинуса, арккосинуса, арктангенса и арккотангенса. Основное внимание здесь нужно уделить на идеально точное воспроизведение определений, так как даже самое маленькое отличие от “идеала” влечет за собой большие ошибки.

Темы 2-4. Функции у=arcsin x, y=arccos x их графики и свойства.

Свойства функций: область определения, область значений, непрерывность, четность и нечетность, возрастание и убывание, экстремумы, наибольшие и наименьшие значения, сохранение знака. Графики функций и их преобразование.

Темы 5-7. Функции у=arctg x, y=arcctg x, их графики и свойства.

Свойства функций: область определения, область значений, непрерывность, четность и нечетность, возрастание и убывание, экстремумы, наибольшие и наименьшие значения, сохранение знака. Графики функций и их преобразование.

Тема 8. Обратные тригонометрические функции, их свойства и графики.

Решение различных заданий, связанных с понятием обратных тригонометрических функций, из вариантов ЕГЭ (группа В и С).

Тема 9. Итоговый контроль.

Итоговая диагностика может быть проведена в виде зачета, виде тестовых заданий, но обязательно дифференцированного характера.

Занятие 1. Определения арксинуса, арккосинуса, арктангенса и арккотангенса.

При решении тригонометрических уравнений простейших (кроме частных случаев) или более сложных неизменно приходишь к формулам корней, в которых есть несколько “магических” слов: арксинус, арккосинус, арктангенс или арккотангенс. Эти четыре слова почти для всех старшеклассников становятся “камнем преткновения”, большинство школьников (в том числе и те, кто потом блестяще сдают математику) не могут точно определить эти функции.

Итак, попробуем разобраться в этих запутанных определениях.

у=arcsin x: у – это число (а не угол!), причем у, такое, что sin у = х. Здесь нужно констатировать еще один факт: х [-1;1].

Продемонстрируем на задачах, как применяется это определение.

№1.

а) arcsin 1/2 =?

Решение: 1/2= х. Значит, мы должны найти такое число у, из отрезка , синус которого равен 1/2. Можно сделать вывод, что у=.

arcsin 1/2 = .

б) arcsin=?

Решение: Рассуждаем аналогично. = х. Значит, мы должны найти такое число у, из отрезка , синус которого равен . Можно сделать вывод, что у= .

arcsin= .

в) arcsin (-)=?

Решение: К этому моменту, почти все старшеклассники (особенно те, которые чуть слабее в знаниях), понимают, что ответ гораздо быстрее найти в учебнике, на первых страницах (есть там такие “замечательные” таблицы). И тут начинаются ошибки. Их надо сразу пресечь, четко повторяя, что у число из отрезка . Для того чтобы найти это число у, можно воспользоваться такой формулой arcsin(-х)= — arcsin х.

Теперь, решение будет гораздо проще.

arcsin (-)= — arcsin = — .

y=arccos x: у – это число (а не угол!), причем у, такое, что cos у = х. Здесь нужно констатировать еще один факт: х [-1; 1].

у=arctg x: у – это число (а не угол!), причем у, такое, что tg у= х. Причем для х здесь ограничений нет.

y=arcctg x: у – это число (а не угол!), причем у , такое, что ctg у= х. Причем для х здесь ограничений нет.

№2.

а) arccos 1/2=?

Решение: 1/2=х. Значит, мы должны найти такое число у, из отрезка , косинус которого равен 1/2. Можно сделать вывод, что у=.

arccos 1/2= .

б) ) arccos=?

Решение: Рассуждаем аналогично. = х. Значит, мы должны найти такое число у, из отрезка, косинус которого равен . Можно сделать вывод, что у= .

arccos= .

в) arccos(-)=?

Решение: Для того чтобы школьники опять не воспользовались таблицами, следует сразу им дать формулу: arсcos(-х) = – arсcos х.

Для вычисления отрицательных значений арктангенса и арккотангенса применимы формулы: arctg(- x) = — arctg x

№ 3.

Вычислить:

а) arctg0

б) arсcos(-1/2)

в) arсctg(-1)

г) arcsin 1

д)

е) arcsin (-0,5)

ж)

№ 4.

Найти область допустимых значений переменной для выражений:

а) arcsin(1-х)

б) arсcos(2-х/2)

в) arcsin(2х+х2)

г) arctg (1-х2)

д)

№5.

Вычислить:

а) sin (arсcos (-1/4))

б) cos (arcctg(-2))

в) sin (2 arcsin 1/3)

г) tg (2 arcsin 1/3).

Решение: а) sin (arсcos (-1/4))=?

Пусть у= arсcos (-1/4). Значит, мы должны найти sin y.

По определению арккосинуса у – это число, из отрезка , косинус которого равен -1/4.

Итак, у= arсcos (-1/4), у, т.е. у может принадлежать I и II четвертям. При этом cos у = -1/4.

Теперь можно уточнить, у принадлежит II четверти, т.к. cos у<0. Используем формулу

sin2y + cos2y =1.

sin2y= 1 — cos2y

siny = ±, т.к. у II ч., то siny>0.

Значит, siny= .

Ответ: siny=.

№6.

Произведите указанные действия:

а) arcsin 3/5 + arcsin 12/13

б) arсcos 7/25 + arсcos 3/5

в) arсctg 5 — arсctg 4

г) arctg4 + arctg 5.

Решение:

Пусть arcsin 3/5 + arcsin 12/13= у, тогда cos у=cos(arcsin 3/5 + arcsin 12/13). Применим формулу косинус суммы и получим:

cos у= cos (arcsin 3/5) cos(arcsin 12/13) – sin(arcsin 3/5) sin(arcsin 12/13)

Вычисляя каждое выражение в отдельности, получим cos у= -16/65, значит у=arсcos(-16/65)

Ответы:

3. а) 0 б) в) г) д) е) ж) 0.

4. а) [ 0;2] б) [ 2;6] в) г) ж)

5. б) в) г) .

6. а) arсcos(-16/65) б) arсcos(-3/5) в) arctg1/21 г) arсctg(-19/9)

Итогом этого занятия должен быть математический диктант с последующей проверкой. Проверка может осуществляться через проецирование с помощью оверхеда, ответы могут быть заранее готовы на дополнительных досках, а также к проверке можно привлечь и учащихся.

Занятия 2-4. Функции у=arcsin x, y=arccos x, их графики и свойства.

Данные занятия следует начинать с понятия обратная функция.

Определение. Пусть каждому значению у Е(f) соответствует только одно значение х D(f), для которого у= f (х). Указанное соответствие у>х задает функцию с областью определения Е(f) и областью значений D(f). Эту функцию называют обратной к функции f (х). Обозначив обратную функцию через g, имеем: если у= f (х), то х = g(у).

Примерами обратных функций могут служить показательная и логарифмическая функции. Для каждой из этих функций всегда можно найти обратную функцию. А вот для функции у=х2 есть обратная функция только при определенных условиях. При каких? (Монотонность функции. y=х2 имеет обратную ей только для х ). Каким свойством обладают графики взаимообратных функций? (Графики взаимно обратных функций симметричны относительно прямой у=х).

Используя эти определения и свойства, построим графики функций у=arcsin x, y=arccos x. Объяснение лучше проводить с помощью ИКТ.

Слайд 1.

С помощью средств анимации построение графика функции у= arcsin х будет выполнено пошагово и наглядно.

Аналогично поступаем и с функцией у= arccos x.

Слайд 2.

Далее необходимо напомнить учащимся о возможных преобразованиях графиков функций и выполнить с классом устную работу.

Устная работа.

  1. Установить соответствие между графиком и формулами.
  2. Слайд 3.

    2. Указать для каждой из данных функции область определения и область значений.

    3. Решить уравнения:

    а) arccos x= 3х+ 3,15

    б) arcsin х= (1/2)х + 1,58

    №1.

    Построить графики функций:

    а) у=2 arccos (х+2) – 2

    б) у= -0.5 arcsin (x-1) +1

    в) y= | 3 arccos (х+1,5)- 5 |

    Это задание может быть выполнено школьниками с помощью таблиц Эльконина–Давыдова с последующей взаимопроверкой. Таблица выглядит следующим образом.

    №2.

    Укажите все точки на оси Ох, являющиеся проекциями точек графика функции:

    Текст задания поставит в тупик многих школьников. Смысл этого задания состоит в том, что процесс нахождения области определения функции совпадает с заданием в этом номере.

    №3.

    Решить уравнение:

    Текст этого задания можно варьировать: найти нули функции, найти абсциссы точек пересечения графиков функций, определить значения х, при которых точки одного графика лежат на графике другой функции.

    №4.

    Найти область определения функции:

    №5.

    Найти область значений функции:

    Текст этого задания можно сформулировать иначе: найти сумму наибольшего и наименьшего значений функции, указать число целых значений функции.

    Ответы:

    2.а) (0;1] б) в) (0;1]

    3.а) 1 б) -1 в) 2

    4. а) б) [0;1/2] в) [2;3)U(3;4]

    5. а) [1;2] б) в) [0;25]

    Задания для самостоятельной работы

    1. Вычислить:

    2. Найти область определения функции:

    3. Найти сумму наибольшего и наименьшего значений функции:

    4. Решить уравнение:

    Занятия 5-7. Функции у=arctg x, y=arcctg x, их графики и свойства.

    Объяснение материала рекомендую вести с помощью ИКТ, проводя сравнительный анализ между функциями у=tg x и у=arctg x, y=ctg x и y=arcctg x. С помощью средств анимации построение графиков функций будет выполнено пошагово и наглядно.

    Слайд 4.

    Слайд 5.

    Далее необходимо напомнить учащимся о возможных преобразованиях графиков функций и выполнить с классом устную работу.

    Устная работа.

    1. Установить соответствие между графиком и формулами:

    Слайд 6.

  3. Для каждой из предложенных функций указать область определения и область значений.
  4. При каком значении а уравнения не имеют решений:

а) arctg x=cos x+ a

б) arcctg x — а = х.

№1.

Построить графики функций:

Это задание может быть выполнено школьниками с помощью таблиц Эльконина–Давыдова с последующей взаимопроверкой. Таблица выглядит следующим образом.

№ 2.

Решить уравнения:

Опыт показывает, что нередко ученик, “берясь” за решение уравнения (впрочем, как и неравенства), концентрирует свое внимание только на поиске преобразований, сводящих исходное уравнение к более простому, забывая при этом, что не каждое преобразование безобидно. Нужно помнить и о свойствах функций, их области определения и области значений. При решении приведенных выше уравнений необходимо обязательно найти ОДЗ.

№ 3.

Найти множество значений функции:

№ 4.

Решить неравенство:

Решение:

в)

Решение данного неравенства опирается на свойства функций y=sin x и y=arctg x . Введем функции y1=sin x-1999 и y2=2arctg x +.

Е(sin x) = [-1; 1], E(y1) =[-2000; -1998]. Это значит, что выражение sin x-1999 < 0 при любых значениях аргумента. Поэтому, выражение 2arctg x +должно принимать неотрицательные значения, т.е. 2arctg x +0.

2arctg x — .

arctg x — .

Так как функция y2=2arctg x + возрастающая, то знак неравенства при дальнейшем решении сохраняется. То есть

Ответ: х.

№ 5.

При каких значениях а уравнение имеет единственный корень:

Ответы:

2. а) 1 б) 0; 2 в) 2; 3

3. а) [0; 2] б) [-1; 0] в) [-3; 0]

4. а) б)

5. а) б) в)

Задания для самостоятельной работы

№ 1.

Вычислить:

№ 2.

Найти множество значений функции:

№ 3.

Решить уравнение или неравенство:

Занятия 8-9. Обратные тригонометрические функции, их свойства и графики.

Эти два занятия я рекомендую провести как практикум, заранее разделив класс на группы. В каждой группе должны быть учащиеся с разной математической подготовкой, тогда работа класса будет более плодотворной и результативной.

Приведу примерный вариант карточек для проведения этого практикума.

Карточка 1.

  1. Построить графики функций:
  2. Вычислить:
  3. Вычислить значения следующих выражений:

Карточка 2.

1.Найти область определения функции:

2. Найти множество значений функции:

3.Найти наименьшее значение функции:

Карточка 3.

  1. Решить уравнения:
  2. Найти сумму х00, если (х00) – решение системы
  3. Решить неравенства:

Карточка 4.

1.Сколько получится числовых промежутков, если из отрезка, определяемого множество значений функции , удалить все целые числа?

2. Для каждого значения параметра а решить неравенство .

Занятие 10-11. Зачет (тест)

В качестве зачетных заданий предлагаются задания из разделов “Задания для самостоятельной работы”. Школьникам заранее дать текст этих заданий, провести консультацию по возникшим вопросам.

См. презентацию.

Арккосинус: определение и обзор — видео и расшифровка урока

Косинус

Косинус (cos) угла прямоугольного треугольника представляет собой тригонометрическую функцию, которую можно определить как отношение стороны, прилежащей к углу, и гипотенузы.

График функции косинуса выглядит так:

Функция косинуса может использоваться для решения задач, связанных с прямоугольным треугольником.Например: скажем, к стене здания прислонена лестница. Вы хотите узнать высоту лестницы, не перемещая ее и не взбираясь по ней. Вы знаете, что основание лестницы находится в 3 футах от здания и что угол лестницы с землей составляет 75°. Вы можете использовать функцию косинуса, чтобы определить длину лестницы.

cos( x ) = смежная / гипотенуза

cos(75) = 3 фута / гипотенуза

гипотенуза = 3 фута / cos75

гипотенуза = 3 / 0.25882

гипотенуза = 11,59 фута

Итак, высота лестницы 11,59 фута.

Арккосинус

Арккосинус является обратной величиной косинуса. Это наиболее полезно при попытке найти меру угла, когда известны две стороны треугольника.

График арккосинуса выглядит так:

Для решения задач с использованием арккосинуса вам понадобится научный калькулятор. У него должна быть кнопка, обычно над кнопкой косинуса, которая выглядит так:

Арккосинус позволяет найти величину угла, если известно отношение прилежащего катета к гипотенузе.

Примеры

Давайте попробуем несколько примеров.

1) Найдите угловую меру Θ в следующем треугольнике:

cos(Θ) = 7 / 13

cos(Θ) = 0,538

Находя арккосинус, получаем:

Θ = 57,4°

2) Посмотрим на пример из предыдущей лестницы. Если бы вы знали, что лестница имеет высоту 7 футов и что она находится на расстоянии 3 футов от стены у земли, вы можете определить угол, который лестница образует с землей, используя арккосинус:

cos( x ) = 3 футы / 7 футов (смежные / гипотенуза)

cos ( x ) = 0.4286

Теперь, чтобы найти меру этого угла, просто найдите арккосинус 0,4286:

x = 64,6°

Как и в случае любых других обратных операций, вы всегда можете проверить свой ответ, выполнив противоположное действие.

Проверьте этот ответ, вычислив cos(64,6°):

cos(64,6) = 0,4286

Итак, угол, который лестница образует с землей, равен 64,6°.

Резюме урока

обратная операция является противоположной исходной операции.Косинусом (cos) угла прямоугольного треугольника называется отношение стороны, прилежащей к углу, и гипотенузы. Арккосинус является обратной функцией функции косинуса. Это означает, что это противоположные функции, и одна компенсирует другую. Арккосинус в основном используется для определения меры угла, когда известны две стороны прямоугольного треугольника. Он находит применение в навигации, технике и других науках.

arccos — функция тригонометрического арккосинуса — Librow — Цифровые ЖК-панели для автомобилей и катеров

Статья 11 — Приложение А.2

1. Определение

Арккосинус является обратной функцией косинуса.

2. Участок

Арккосинус — это монотонная функция, заданная в диапазоне [−1, 1]. Его участок изображен ниже на рис. 1 .

Рис. 1. График функции арккосинуса y = arccos x .

Кодовый домен функции ограничен диапазоном [0, π].

3. Личности

Дополнительный угол:

arcsinx + arccosx = π/2

и как следствие:

arccos sin φ = π/2 — φ

Отрицательный аргумент:

arccos(−x) = π − arccosx

Обратный аргумент:

arcos(1/x) = arcsecx

Сумма и разница:

arccosx + arccosy = arccos{xy − √[(1 − x 2 )(1 − y 2 )]}
arccosx − arccosy = arccos{xy + √[(1 − x 2 )(1 − у 2 )]}

Некоторые значения аргумента:

√ (2 — √2) / 2 1 / √2 √ (10 + 2√5) / 4
аргумент
Аргумент x

9
Значение Arccos x
0 π / 2
(√6 — √2) / 4 5π / 12
(√5 — 1) / 4 2π / 5
3π / 8
1/2 π / 3
√ (10 — 2√ 5) / 4 3π / 10
π / 4
(√5 + 1) / 4 π / 5
√3 / 2 π / 6
√ (2 + √2) / 2 π / 8
π / 10
(√6 + √2) /4 π/12
1 0
Таблица 1. Арккосинус для некоторых значений аргументов.

4. Опора

Тригонометрическая функция арккосинуса arccos действительного аргумента поддерживается бесплатно версия калькулятора Librow.

Тригонометрическая функция арккосинуса arccos комплексного аргумента поддерживается профессиональный версия калькулятора Librow.

5. Как пользоваться

Для вычисления арккосинуса числа:

Для вычисления арккосинуса текущего результата:

Чтобы вычислить арккосинус числа x в памяти:

Arctg 2 равно.Арксинус, арккосинус

Функции sin, cos, tg и ctg всегда сопровождаются арксинусом, арккосинусом, арктангенсом и арктангенсом. Одно является следствием другого, и пары функций одинаково важны для работы с тригонометрическими выражениями.

Рассмотрите рисунок единичной окружности, на котором графически отображаются значения тригонометрических функций.

Если вычислить дуги OA, arcos OC, arctg DE и arcctg MK, то все они будут равны значению угла α. Приведенные ниже формулы отражают связь между основными тригонометрическими функциями и соответствующими им дугами.

Чтобы лучше понять свойства арксинуса, необходимо рассмотреть его функцию. График имеет вид асимметричной кривой, проходящей через центр координат.

Арксинус свойства:

Если сравнить графики sin и arcsin , для двух тригонометрических функций можно найти общие закономерности.

Арккосинус

Arccos числа а есть значение угла α, косинус которого равен а.

Кривая y = arcos x зеркально отражает график arcsin x, с той лишь разницей, что она проходит через точку π/2 на оси OY.

Рассмотрим функцию арккосинуса более подробно:

  1. Функция определена на сегменте [-1; 1].
  2. ОДЗ для arccos -.
  3. График полностью расположен в I и II четвертях, а сама функция не является ни четной, ни нечетной.
  4. Y = 0 для х = 1.
  5. Кривая уменьшается по всей длине. Некоторые свойства арккосинуса такие же, как у функции косинуса.

Некоторые свойства арккосинуса такие же, как у функции косинуса.

Возможно, такое «детальное» изучение «арок» школьникам покажется излишним. Однако в противном случае некоторые элементарные типовые задания ЕГЭ могут завести студентов в тупик.

Упражнение 1. Укажите функции, показанные на рисунке.

Ответ: рис. 1 — 4, рис. 2 — 1.

В этом примере упор делается на мелочи. Обычно студенты очень невнимательно относятся к построению графиков и внешнему виду функций. Действительно, зачем запоминать тип кривой, если ее всегда можно построить по рассчитанным точкам. Не забывайте, что в тестовых условиях время, потраченное на рисование простой задачи, потребуется для решения более сложных задач.

Арктангенс

Arctg число а такое значение угла α, что его тангенс равен а.

Если рассматривать график арктангенса, то можно выделить следующие свойства:

  1. Граф бесконечен и определен на интервале (- ∞; + ∞).
  2. Арктангенс — нечетная функция, поэтому арктангенс (- х) = — арктангенс х.
  3. Y = 0 при х = 0.
  4. Кривая поднимается по всей области определения.

Вот краткий сравнительный анализ tg x и arctan x в виде таблицы.

Арккотангенс

Arcctg числа a — принимает такое значение α из интервала (0; π), что его котангенс равен a.

Свойства функции арккотангенса:

  1. Интервал определения функции бесконечен.
  2. Диапазоном допустимых значений является интервал (0; π).
  3. F (x) не является ни четным, ни нечетным.
  4. График функции убывает по всей длине.

Сравнить ctg x и arctan x очень легко, нужно просто нарисовать две картинки и описать поведение кривых.

Задание 2. Соотнесите график и форму записи функции.

Логично, что графики показывают, что обе функции возрастают. Следовательно, оба рисунка отображают некоторую функцию arctg. Из свойств арктангенса известно, что y = 0 при x = 0,

Ответ: рис.1 — 1, рис. 2 — 4.

Тригонометрические тождества arcsin, arcos, arctg и arcctg

Ранее мы уже определили связь между арками и основными функциями тригонометрии. Эту зависимость можно выразить рядом формул, позволяющих выразить, например, синус аргумента через его арксинус, арккосинус или наоборот. Знание таких тождеств может оказаться полезным при решении конкретных примеров.

Также существуют коэффициенты для arctg и arcctg:

Другая полезная пара формул устанавливает значение суммы значений arcsin и arcos, а также arcctg и arcctg одного и того же угла.

Примеры решения задач

Задачи тригонометрии условно можно разделить на четыре группы: вычислить числовое значение конкретного выражения, построить график этой функции, найти ее область определения или ОДЗ и выполнить аналитические преобразования для решения примера.

При решении первого типа задач необходимо придерживаться следующего плана действий:

При работе с графиками функций главное знать их свойства и вид кривой. Таблицы тождеств нужны для решения тригонометрических уравнений и неравенств. Чем больше формул запоминает ученик, тем легче ему найти ответ на задание.

Допустим, на экзамене вам нужно найти ответ на уравнение вида:

Если правильно преобразовать выражение и привести его к нужному виду, то решить его очень просто и быстро. Во-первых, давайте переместим arcsin x в правую часть равенства.

Если вспомнить формулу arcsin(sin α) = α , то поиск ответов можно свести к решению системы двух уравнений:

Ограничение на модель x возникло опять же из свойств arcsin: ОДЗ для x[-1; 1].При а ≠ 0 часть системы представляет собой квадратное уравнение с корнями х1 = 1 и х2 = — 1/а. При а = 0 х будет равен 1.

Примечание 1

Таблица Брадиса — таблица, позволяющая с высокой точностью вычислять значения арктангенсов и других тригонометрических функций.

Чтобы пользоваться таблицей Брадиса, ищите угол в градусах в крайнем левом столбце для синуса (для косинуса в соответствующем столбце справа), а затем в верхней строке минуты. На пересечении строки со столбцом находится искомое значение.

При необходимости найти значения обратных тригонометрических функций таблица Брадиса используется наоборот. Например, ищут числовое значение в таблице арктангенсов и тангенсов и определяют по нему, в какой строке градусов и столбце минут оно находится.

Таким образом, таблицу Брадиса можно использовать не только для поиска обычных тригонометрических функций, но и как таблицу арккосинуса и арксинуса, арктангенса и арккотангенса.

В начале этой статьи находится таблица значений arcsin и arccos, а ближе к концу — таблица значений arctg и arcctg.

Таблица Брадиса: arcsin, arccos, cos и таблица sin

Рис. 1. Таблица Брадиса представляет собой таблицу значений арксинуса и арккоса. Author24 — онлайн обмен студенческими работами

Таблица значений арктангенсов и арккотангенсов, тангенсов и котангенсов

Рисунок 4. Таблица Брадиса: таблица значений арктангенсов и арктангенсов арктангенсов. Author24 — онлайн обмен студенческими работами

Определение и обозначения

Арксинус (y = arcsin x) есть функция обратного синуса (x = sin y -1 ≤ x ≤ 1 и множество значений -π/2 ≤ y ≤ π/2.
sin (arcsin x) = x ;
arcsin(sin х) = х.

Арксинус иногда обозначается следующим образом:
.

График функции арксинуса

График функции y = arcsin x

График арксинуса получается из графика синуса путем перестановки осей абсцисс и ординат.Для устранения неоднозначности диапазон значений ограничен интервалом, на котором функция монотонна. Это определение называется главным значением арксинуса.

Арккосинус, arccos

Определение и обозначения

Арккосинус (y = arccos x) — функция, обратная косинусу (x = cos y). Он имеет область действия -1 ≤ x ≤ 1 и множество значений 0 ≤ y ≤ π.
cos(arccos x) = x ;
arccos(cos x) = x.

Арккосинус иногда обозначается следующим образом:
.

График функции арккосинуса


График функции y = arccos x

График арккосинуса получается из графика косинуса путем перестановки осей абсцисс и ординат. Для устранения неоднозначности диапазон значений ограничен интервалом, на котором функция монотонна. Это определение называется главным значением арккосинуса.

Четность

Функция арксинуса нечетная:
arcsin (- x) = arcsin (-sin arcsin x) = arcsin (sin (-arcsin x)) = — arcsin x

Функция арксинуса не четный или нечетный:
arccos(- x) = arccos(-cos arccos x) = arccos(cos(π-arccos x)) = π — arccos x ≠ ± arccos x

Свойства — экстремумы, возрастание, уменьшение

Функции арксинуса и арккосинуса непрерывны в своей области определения (см. доказательство непрерывности).Основные свойства арксинуса и арксинуса представлены в таблице.

у = arcsin х у = arccos х
Объем и преемственность — 1 ≤ x ≤ 1 — 1 ≤ х ≤ 1
Диапазон значений
Увеличение, уменьшение монотонно возрастает монотонно убывает
Высшее
Минимум
Нули, у = 0 х = 0 х = 1
Точки пересечения с осью у, х = 0 у = 0 у = π/2

Таблица арксинусов и арккосинусов

В этой таблице приведены значения арксинусов и арксинусов, в градусах и радианах, для некоторых значений аргумента.

x arcsin x арккос х
град. рад. град. рад.
— 1 — 90° 180° π
— 60° 150°
— 45° 135°
— 30° 120°
0 0 90°
30° 60°
45° 45°
60° 30°
1 90° 0

≈ 0,7071067811865476
≈ 0,8660254037844386

Формулы

См. также: Вывод формул для обратных тригонометрических функций

Сумма и разностные формулы


в или

и

и


в и


по

и

и

и


по

на

по


по

при

выражений логарифма, Комплексные числа

См. также: Вывод формул.

Выражения через гиперболические функции

Производные

;
.
См. Производные арксинуса и арккосинуса производные >>>

Производные высших порядков :
,
где — полином степени. Определяется по формулам:
;
;
.

См. Вывод высших производных арксинуса и арккосинуса >>>

Интегралы

Подстановка x = sin t . Интегрируем по частям, учитывая, что -π/2 ≤ t ≤ π/ 2, cos т ≥ 0:
.

Выразим арккосинус через арксинус:
.

Расширение серии

Для | х |1 происходит следующее разложение:
;
.

Обратные функции

Обратными значениями арксинуса и арккосинуса являются соответственно синус и косинус.

Следующие формулы справедливы во всей области:
sin(arcsin x) = x
cos(arccos x) = x .

Следующие формулы справедливы только на множестве значений арксинуса и арксинуса:
arcsin(sin x) = x at
arccos(cos x) = x at.

Каталожные номера:
И.Н. Бронштейн, К.А. Семендяев, Справочник по математике для инженеров и студентов технических вузов, «Лань», 2009.

См. также:

Что такое арксинус, арккосинус? Что такое арктангенс, арккотангенс?


Внимание!
В Спецразделе 555 имеются дополнительные материалы
.
Для тех, кому «не очень…»
И для тех, кто «очень…»)

К понятиям арксинус, арккосинус, арктангенс, арккотангенс обучающихся людей насторожены.Он не понимает этих терминов и, следовательно, не доверяет этой милой семье.) А зря. Это очень простые понятия. Что, кстати, здорово облегчает жизнь знающему человеку при решении тригонометрических уравнений!

Сомневаетесь в простоте? Напрасно. ) Прямо здесь и сейчас вы в этом убедитесь.

Конечно, для понимания неплохо бы знать, что такое синус, косинус, тангенс и котангенс. Да, их табличные значения для некоторых углов… По крайней мере, в самых общих чертах. Тогда и здесь проблем не будет.

Итак, мы удивляемся, но помните: арксинус, арккосинус, арктангенс и арккотангенс — это всего лишь некоторые углы. Не больше, не меньше. Есть угол, допустим 30°. И есть угол угловых углов 0,4. или arctg (-1,3). Углы бывают всякие.) Просто углы можно записывать по разному. Вы можете написать угол в градусах или радианах.А можно — через его синус, косинус, тангенс и котангенс…

Что означает выражение

угловой синус 0,4?

Это угол, синус которого равен 0,4 ! Да Да. В этом смысл арксинуса. Специально повторю: arcsin 0,4 — это угол, синус которого равен 0,4.

И все.

Чтобы надолго удержать эту простую мысль в голове, я даже приведу расшифровку этого страшного термина — арксинус:

дуга sin 0,4
угол, синус которого равен 0. 4

Как написано, так и слышно.) Почти. Приставка дуга означает дуга (слово дуга знаете?) древние люди вместо углов использовали дуги, но сути дела это не меняет. Запомните эту элементарную расшифровку математического термина! Причем для арккосинуса, арктангенса и арккотангенса расшифровка отличается только названием функции.

Что такое arccos 0.8?
Это угол, косинус которого равен 0.8.

Что такое arctg (-1,3)?
Это угол, тангенс которого равен -1,3.

Что такое arcctg 12?
Это угол, котангенс которого равен 12.

Кстати, такая элементарная расшифровка позволяет избежать эпических ляпов.) Например, вполне солидно выглядит выражение arccos1,8. Приступаем к расшифровке: arccos1,8 — это угол, косинус которого равен 1,8… Доп-Дап!? 1,8!? Косинус не может быть больше единицы!!!

Правильно.Выражение arccos1,8 бессмысленно. И написание такого выражения в каком-нибудь ответе сильно позабавит экзаменатора. )

Элементарно, как видите.) У каждого угла свои личные синус и косинус. И почти у каждого свой тангенс и котангенс. Следовательно, зная тригонометрическую функцию, можно записать и сам угол. Для этого предназначены арксинусы, арккосинусы, арктангенсы и арккотангенсы. Далее я буду называть все это семейство уменьшительно-ласкательным — арок. Чтобы печатать меньше.)

Внимание! Элементарная словесная и сознательная расшифровка арок позволяет спокойно и уверенно решать самые разные задачи. А в необычных задачах только она и спасает.

Можно ли переключиться с арок на обычные градусы или радианы? — слышу осторожный вопрос.)

Почему бы и нет!? Легкий. И можно туда и обратно. Более того, иногда это необходимо делать. Арки вещь простая, но без них как-то спокойнее, правда?)

Например: что такое arcsin 0.5?

Напоминаем расшифровку: arcsin 0.5 — это угол, синус которого равен 0.5. Теперь включаем голову (или гугл)) и вспоминаем под каким углом синус 0,5? Синус равен 0,5 y угол 30 градусов . .. Вот и все: arcsin 0,5 угол 30°. Можно смело писать:

arcsin 0,5 = 30°

Или, точнее, в радианах:

Все, можно забыть об арксинусе и продолжить работу с обычными градусами или радианами.

Если бы ты понял что такое арксинус, арккосинус… Что такое арктангенс, арккотангенс… С таким монстром можно легко разобраться, например.)

Несведущий человек отшатнется в ужасе, да…) запомните расшифровку: арксинус — это угол, синус которого равен… И так далее. Если знающий человек знает еще и таблицу синусов… Таблица косинусов. Таблица тангенсов и котангенсов, тут вообще проблем нет!

Достаточно понять, что:

расшифрую, т.е.е. Переведу формулу словами: угол, тангенс которого равен 1 (arctg1) это угол 45°. Или, что равно единице, Пи/4. Аналогично:

и все… Замените все арки значениями в радианах, все уменьшится, осталось посчитать сколько будет 1+1. Это будет 2.) Это правильный ответ.

Вот как можно (и нужно) переключаться с арксинусов, арккосинусов, арктангенсов и арккотангенсов на обычные градусы и радианы.Это сильно упрощает страшные примеры!

Часто в таких примерах внутри арок отрицательных значений. Например, arctg (-1,3) или arccos (-0,8)… это не проблема. Вот несколько простых формул для перехода от отрицательных значений к положительным:

Вам нужно, скажем, определить значение выражения:

Это можно решить с помощью тригонометрического круга, но вы не хотите его рисовать. Ну ладно. Переходя от отрицательных значений внутрь арккосинуса к положительных по второй формуле:

Внутри арккосинуса справа уже положительное значение .Что

вы просто должны знать. Осталось подставить вместо арккосинуса радианы и вычислить ответ:

Вот и все.

Ограничения на арксинус, арккосинус, арктангенс, арккотангенс.

Проблемы с примерами 7-9? Ну да, тут есть какая-то хитрость.)

Все эти примеры с 1 по 9 тщательно разобраны в разделе 555. Что, как и почему. Со всеми секретными ловушками и уловками. Плюс способы кардинально упростить решение.Кстати, в этом разделе много полезной информации и практических советов по тригонометрии в целом. И не только тригонометрия. Очень помогает.

Если вам нравится этот сайт…

Кстати, у меня для вас есть еще парочка интересных сайтов.)

Вы можете попрактиковаться в решении примеров и узнать свой уровень. Тестирование с мгновенной проверкой. Учимся — с интересом!)

вы можете ознакомиться с функциями и производными.

Проще говоря, это овощи, сваренные на воде по особому рецепту. Я буду рассматривать два исходных компонента (овощной салат и вода) и готовый результат — борщ. Геометрически это можно представить как прямоугольник, одна сторона которого представляет салат, а другая — воду. Сумма этих двух сторон будет представлять собой борщ. Диагональ и площадь такого «борщевого» прямоугольника являются чисто математическими понятиями и никогда не используются в рецептах борща.


Как математически салат и вода превращаются в борщ? Как сумма двух отрезков может превратиться в тригонометрию? Чтобы понять это, нам нужны линейные угловые функции.


Вы не найдете ничего о линейных угловых функциях в учебниках по математике. Но без них не может быть математики. Законы математики, как и законы природы, работают вне зависимости от того, знаем мы об их существовании или нет.

Линейные угловые функции являются законами сложения. Посмотрите, как алгебра превращается в геометрию, а геометрия превращается в тригонометрию.

Можно ли обойтись без линейных угловых функций? Это возможно, потому что математики до сих пор обходятся без них. Хитрость математиков заключается в том, что они всегда рассказывают нам только о тех задачах, которые сами умеют решать, и никогда не говорят о тех задачах, которые не могут решить. Смотреть. Если мы знаем результат сложения и один член, мы используем вычитание, чтобы найти другой член. Все. Других задач мы не знаем и не можем их решить.Что делать, если мы знаем только результат сложения и не знаем обоих членов? В этом случае результат сложения необходимо разложить на два слагаемых с помощью линейных функций угла. Затем мы сами выбираем, каким может быть одно слагаемое, а линейные угловые функции показывают, каким должно быть второе слагаемое, чтобы результат сложения был именно таким, какой нам нужен. Таких пар терминов может быть бесконечное множество. В повседневной жизни мы прекрасно обходимся без разложения суммы; вычитания нам достаточно.Но в научном исследовании законов природы разложение суммы на члены может быть очень полезным.

Еще один закон сложения, о котором математики не любят говорить (еще одна их уловка), требует, чтобы члены имели одинаковые единицы измерения. Для салата, воды и борща это могут быть единицы веса, объема, стоимости или единицы измерения.

На рисунке показаны два уровня различий по математике. Первый уровень — это различия в поле цифр, которые обозначаются а , б , в … Этим занимаются математики. Второй уровень — это разности площадей единиц измерения, которые показаны в квадратных скобках и обозначены буквой U … Этим занимаются физики. Мы можем понять третий уровень — различия площади описываемых предметов. Разные объекты могут иметь одинаковое количество одинаковых единиц. Насколько это важно, мы можем видеть на примере борщевой тригонометрии. Если к одному и тому же обозначению единиц измерения разных объектов добавить нижние индексы, то можно точно сказать, какое математическое значение описывает тот или иной объект и как оно изменяется во времени или в связи с нашими действиями.Буквой W обозначу воду, буквой S обозначу салат и буквой B — борщ. Вот как будут выглядеть линейные угловые функции для борща.

Если взять часть воды и часть салата, вместе они превратятся в одну порцию борща. Вот предлагаю вам отдохнуть от борща и вспомнить свое далекое детство. Помните, как нас учили складывать зайчиков и уточек? Нужно было найти, сколько животных будет.Что же тогда нас учили делать? Нас учили отделять единицы от чисел и складывать числа. Да, любой номер можно добавить к любому другому номеру. Это прямой путь к аутизму современной математики — мы не понимаем чего, непонятно почему, и очень плохо понимаем, как это соотносится с реальностью, ведь из трех уровней различия математика оперирует только одним. Правильнее было бы научиться переключаться с одной единицы измерения на другую.

Кроликов, уток и животных можно считать поштучно.Одна общая единица измерения для разных объектов позволяет складывать их вместе. Это детская версия проблемы. Давайте рассмотрим аналогичную проблему для взрослых. Что произойдет, если вы добавите кроликов и деньги? Здесь есть два возможных решения.

Первый вариант … Определяем рыночную стоимость зайчиков и прибавляем ее к имеющейся сумме денег. Мы получили общую стоимость нашего богатства в денежном выражении.

Второй вариант … К количеству имеющихся у нас банкнот можно добавить количество зайчиков.Мы получим количество движимого имущества в штуках.

Как видите, один и тот же закон сложения дает разные результаты. Все зависит от того, что именно мы хотим узнать.

Но вернемся к нашему борщу. Теперь мы можем увидеть, что будет происходить при разных значениях угла линейных функций угла.

Угол равен нулю. У нас есть салат, но нет воды. Мы не можем сварить борщ. Количество борща тоже ноль. Это вовсе не означает, что ноль борща равен нулю воды.Нулевой борщ может быть нулевым салатом (прямой угол).


Лично для меня это главное математическое доказательство того, что. Ноль не меняет число при добавлении. Это связано с тем, что само сложение невозможно, если есть только один член, а второй член отсутствует. Вы можете относиться к этому как угодно, но помните — все математические операции с нулем были придуманы самими математиками, так что отбросьте свою логику и тупо впихивайте придуманные математиками определения: «деление на ноль невозможно», «любое число, умноженное на ноль равно нулю», «за нулевой точкой» и прочая чепуха. Достаточно один раз вспомнить, что ноль не является числом, и у вас больше никогда не возникнет вопроса, является ли ноль натуральным числом или нет, потому что такой вопрос вообще теряет всякий смысл: как можно считать число, которое не является числом . Это все равно что спрашивать, какого цвета должен быть невидимый цвет. Добавление нуля к числу похоже на рисование несуществующей краской. Мы помахали сухой кистью и сказали всем, что «нарисовали». Но я немного отвлекся.

Угол больше нуля, но меньше сорока пяти градусов.У нас много салата, но мало воды. В итоге получаем густой борщ.

Угол сорок пять градусов. У нас поровну воды и салата. Это идеальный борщ (да простят меня повара, это просто математика).

Угол больше сорока пяти градусов, но меньше девяноста градусов. У нас много воды и мало салата. У вас получится жидкий борщ.

Прямой угол. У нас есть вода. От салата остались только воспоминания, так как продолжаем отмерять угол от линии, которая когда-то стояла за салатом. Мы не можем сварить борщ. Количество борща равно нулю. В таком случае держитесь и пейте воду, пока есть)))

Вот. Что-то вроде этого. Я могу рассказать здесь и другие истории, которые здесь более чем уместны.

У двух друзей были свои доли в общем деле. После убийства одного из них все досталось другому.

Появление математики на нашей планете.

Все эти истории на языке математики рассказываются с использованием функций линейного угла.Как-нибудь в другой раз я покажу вам реальное место этих функций в структуре математики. А пока вернемся к тригонометрии борща и рассмотрим проекции.

суббота, 26 октября 2019 г.

среда, 7 августа 2019 г.

Завершая разговор, можно рассмотреть бесконечное множество. Это дало то, что понятие «бесконечность» действует на математиков, как удав на кролика. Трепетный ужас перед бесконечностью лишает математиков здравого смысла.Вот например:

Первоисточник находится. Альфа означает действительное число. Знак равенства в вышеприведенных выражениях говорит о том, что если к бесконечности прибавить число или бесконечность, ничего не изменится, в результате получится та же бесконечность. Если взять в качестве примера бесконечное множество натуральных чисел, то рассматриваемые примеры можно представить в таком виде:

Для наглядного доказательства их правильности математики придумали множество различных способов. Лично я смотрю на все эти способы как на пляски шаманов с бубнами.По существу, все они сводятся к тому, что либо часть комнат не занята и въезжают новые гости, либо часть посетителей выбрасывается в коридор, чтобы освободить место для гостей (очень по-человечески). Свой взгляд на такие решения я изложил в виде фантастического рассказа о Блондинке. На чем основываются мои рассуждения? Перемещение бесконечного числа посетителей занимает бесконечное количество времени. После того, как мы освободили первую комнату для гостя, один из посетителей всегда будет ходить по коридору из своей комнаты в следующую до скончания века. Конечно, фактор времени можно тупо не учитывать, но это уже будет из разряда «закон для дураков не писан». Все зависит от того, что мы делаем: подгоняем реальность под математические теории или наоборот.

Что такое «бесконечный отель»? Бесконечная гостиница – это гостиница, в которой всегда есть любое количество свободных мест, независимо от того, сколько номеров занято. Если все комнаты в бесконечном коридоре для посетителей заняты, то есть еще один бесконечный коридор с гостевыми комнатами. Таких коридоров будет бесконечное множество.Более того, «бесконечный отель» имеет бесконечное количество этажей, бесконечное количество зданий, бесконечное количество планет, бесконечное количество вселенных, созданных бесконечным количеством Богов. Математики, однако, не в состоянии дистанцироваться от обыденных житейских проблем: Бог-Аллах-Будда всегда один, гостиница одна, коридор один. Здесь математики пытаются манипулировать порядковыми номерами гостиничных номеров, убеждая нас в том, что можно «засунуть вещи внутрь».

Я продемонстрирую вам логику своих рассуждений на примере бесконечного множества натуральных чисел. Для начала вам нужно ответить на очень простой вопрос: сколько существует множеств натуральных чисел — один или много? правильный ответ на этот вопрос, так как мы сами придумали числа, в Природе чисел не существует. Да, Природа превосходно умеет считать, но для этого она использует другие математические средства, не знакомые нам. Как думает Природа, я расскажу ты в другой раз.Поскольку мы изобрели числа, мы сами будем решать, сколько существует наборов натуральных чисел. Рассмотрим оба варианта, как и подобает настоящему ученому.

Вариант первый. «Дайте нам» единый набор натуральных чисел, который безмятежно лежит на полке. Берем этот набор с полки. Все, других натуральных чисел на полке нет и взять их неоткуда. Мы не можем добавить его в этот набор, так как он у нас уже есть. А если очень хочется? Без проблем. Можем взять одну из уже взятого набора и вернуть на полку.После этого мы можем взять единицу с полки и добавить ее к тому, что у нас осталось. В результате мы снова получаем бесконечное множество натуральных чисел. Все наши манипуляции можно записать так:

Я записал действия в алгебраической записи и в записи, используемой в теории множеств, с подробным перечислением элементов множества. Нижний индекс указывает на то, что у нас есть один-единственный набор натуральных чисел. Оказывается, множество натуральных чисел останется неизменным, только если из него вычесть и прибавить ту же единицу.

Второй вариант. У нас на полке много разных бесконечных наборов натуральных чисел. Подчеркну — РАЗНЫЕ, несмотря на то, что практически неотличимы. Берем один из этих наборов. Затем мы берем одно из другого набора натуральных чисел и добавляем его к уже взятому нами набору. Мы можем даже сложить два набора натуральных чисел. Вот что мы получаем:

Нижние индексы «один» и «два» указывают на принадлежность этих предметов к разным наборам. Да, если вы добавите единицу к бесконечному множеству, результатом также будет бесконечное множество, но оно не будет таким же, как исходное множество. Если мы добавим еще одно бесконечное множество к одному бесконечному множеству, результатом будет новое бесконечное множество, состоящее из элементов первых двух множеств.

Многие натуральные числа используются для счета так же, как линейка для измерений. Теперь представьте, что вы прибавили к линейке один сантиметр. Это будет уже другая строка, не равная исходной.

Вы можете принять или не принять мои рассуждения — это ваше личное дело. Но если вы когда-нибудь столкнетесь с математическими проблемами, подумайте, не идете ли вы по пути ложных рассуждений, протоптанному поколениями математиков.Ведь занятия математикой, прежде всего, формируют в нас устойчивый стереотип мышления, а уже потом добавляют нам умственных способностей (или наоборот, лишают свободы мысли).

позг.ру

воскресенье, 4 августа 2019 г.

Писал постскриптум к статье о и увидел в Википедии этот замечательный текст:

Читаем: «…богатая теоретическая база математики Вавилона не носила целостного характера и сводилась к набору разрозненных приемов, лишены единой системы и доказательной базы.

Ничего себе! Какие мы умные и как хорошо умеем видеть недостатки других. Тяжело ли нам смотреть на современную математику в том же контексте? Немного перефразируя вышеприведенный текст, лично у меня получилось следующее:

богатая теоретическая база современной математики не является целостной и сводится к набору разрозненных разделов, лишенных общей системы и доказательной базы.

Не буду далеко ходить в подтверждение своих слов — у нее язык и условности, отличные от язык и соглашения многих других разделов математики.Одни и те же имена в разных областях математики могут иметь разное значение. Я хочу посвятить целую серию публикаций наиболее очевидным промахам современной математики. До скорой встречи.

суббота, 3 августа 2019 г.

Как вы подразделяете набор? Для этого необходимо ввести новую единицу измерения, которая присутствует у некоторых элементов выбранного набора. Давайте посмотрим на пример.

Пусть у нас будет множество И , состоящих из четырех человек. Этот набор образован на основе «людей». Обозначим элементы этого набора буквами и , нижний индекс с цифрой будет обозначать порядковый номер каждого человека в этом наборе.Введем новую единицу измерения «пол» и обозначим ее буквой b … Поскольку половые признаки присущи всем людям, умножим каждый элемент множества И на пол b … Примечание что теперь наше множество «людей» стало множеством «людей с половыми признаками». После этого мы можем разделить половые признаки на мужские bm и женские bw половые признаки. Теперь мы можем применить математический фильтр: мы выбираем один из этих половых признаков, независимо от того, мужской он или женский.Если он есть у человека, то умножаем на единицу, если такого признака нет, умножаем на ноль. А дальше применяем обычную школьную математику. Посмотрите, что произошло.

После умножения, сокращения и перестановки мы получили два подмножества: подмножество мужчин Bm и подмножество женщин Bw . .. Примерно так же думают математики, когда применяют теорию множеств на практике. Но они не посвящают нас в подробности, а дают готовый результат — «множество людей состоит из подмножества мужчин и подмножества женщин.Естественно, у вас может возникнуть вопрос, насколько правильно применена математика в приведенных выше преобразованиях? Смею вас уверить, на самом деле преобразования были произведены правильно, достаточно знать математические основы арифметики, булевой алгебры и других разделов математики. Что это такое? Об этом я расскажу как-нибудь в другой раз.

Что касается суперсетов, то вы можете объединить два набора в один суперсет, выбрав единицу измерения, которая присутствует у элементов этих двух наборов.

Как как видите, единицы и общая математика делают теорию множеств делом прошлого.Признаком того, что с теорией множеств не все в порядке, является то, что математики придумали свой собственный язык и обозначения для теории множеств. Математики действовали, как когда-то шаманы. Только шаманы умеют «правильно» применять свои «знания». Они учат нас этому «знанию».

Наконец, я хочу показать вам, как математики манипулируют с.

понедельник, 7 января 2019 г.

В пятом веке до нашей эры древнегреческий философ Зенон Элейский сформулировал свои знаменитые апории, самой известной из которых является апория «Ахиллес и черепаха».Вот как это звучит:

Допустим, Ахилл бежит в десять раз быстрее черепахи и отстает от нее на тысячу шагов. За время, которое потребуется Ахиллесу, чтобы пробежать это расстояние, черепаха проползет сто шагов в том же направлении. Когда Ахиллес пробегает сто шагов, черепаха проползает еще десять шагов и так далее. Процесс будет продолжаться бесконечно, Ахиллес никогда не догонит черепаху.

Это рассуждение стало логическим шоком для всех последующих поколений.Аристотель, Диоген, Кант, Гегель, Гильберт… Все они, так или иначе, считали апории Зенона. Потрясение было настолько сильным, что «. .. дискуссии продолжаются и в настоящее время, научное сообщество еще не успело прийти к единому мнению о сущности парадоксов… математический анализ, теория множеств, новые физико-философские подходы привлекались к изучению вопроса, ни один из них не стал общепринятым решением вопроса… » [Википедия, Апория Зенона»].Все понимают, что их дурят, но никто не понимает, в чем заключается обман.

С точки зрения математики Зенон в своей апории наглядно показал переход от величины к. Этот переход включает в себя применение вместо констант. Насколько я понимаю, математический аппарат для применения переменных единиц измерения либо еще не разработан, либо он не применялся к апориям Зенона. Применение нашей обычной логики приводит нас в ловушку. Мы по инерции мышления применяем постоянные единицы времени к обратным.С физической точки зрения это выглядит как замедление времени, пока оно полностью не остановится в тот момент, когда Ахиллес окажется на одном уровне с черепахой. Если время остановится, Ахилл больше не сможет догнать черепаху.

Если перевернуть привычную логику, все становится на свои места. Ахиллес бежит с постоянной скоростью. Каждый последующий отрезок его пути в десять раз короче предыдущего. Соответственно время, затраченное на его преодоление, в десять раз меньше предыдущего. Если применить к этой ситуации понятие «бесконечность», то правильно будет сказать «Ахиллес бесконечно быстро догонит черепаху».

Как избежать этой логической ловушки? Оставайтесь в постоянных единицах времени и не возвращайтесь назад. На языке Зенона это выглядит так:

За время, за которое Ахиллес пробежит тысячу шагов, черепаха проползет сто шагов в том же направлении.За следующий промежуток времени, равный первому, Ахиллес пробежит еще тысячу шагов, а черепаха проползет сто шагов.Теперь Ахилл опережает черепаху на восемьсот шагов.

Это подход адекватно описывает действительность без каких-либо логических парадоксов.Но это не полное решение проблемы. Утверждение Эйнштейна о непреодолимости скорости света очень похоже на апорию Зенона «Ахиллес и черепаха». Нам еще предстоит изучить, переосмыслить и решить эту проблему. И решение надо искать не в бесконечно больших числах, а в единицах измерения.

Еще одна интересная апория Зенона рассказывает о летящей стреле:

Летящая стрела неподвижна, так как в каждый момент времени она покоится, а так как она покоится в каждый момент времени, то она всегда покоится.

В этой апории логический парадокс преодолевается очень просто — достаточно уточнить, что в каждый момент времени летящая стрела покоится в разных точках пространства, что, собственно, и есть движение. Здесь следует отметить еще один момент. По единственной фотографии автомобиля на дороге невозможно определить ни факт его движения, ни расстояние до него. Для определения факта движения автомобиля необходимы две фотографии, сделанные из одной и той же точки в разное время, но по ним нельзя определить расстояние.Для определения расстояния до автомобиля нужны две фотографии, сделанные из разных точек пространства в одно и то же время, но по ним нельзя определить факт движения (конечно, вам еще нужны дополнительные данные для расчетов, тригонометрия вам в помощь) . На что я хочу обратить особое внимание, так это на то, что две точки во времени и две точки в пространстве — это разные вещи, которые не следует путать, потому что они предоставляют разные возможности для исследования.
Я покажу вам процесс на примере.Выбираем «красное тело в прыщике» — это наше «целое». При этом мы видим, что эти вещи с дужкой, а дужек нет. После этого выделяем часть «целого» и формируем набор «с бантиком». Именно так шаманы кормят себя, привязывая свою теорию множеств к реальности.

А теперь немного пакостим. Возьмите «тело в пупырышке с бантиком» и объедините эти «целые» по цвету, выделив красные элементы. У нас много «красного». Теперь вопрос на заполнение: получившиеся наборы «с бантиком» и «красный» это один и тот же набор или это два разных набора? Только шаманы знают ответ.Точнее сами ничего не знают, но как говорится, так и будет.

Этот простой пример показывает, что теория множеств совершенно бесполезна, когда дело доходит до реальности. В чем секрет? У нас сформировался набор «красное тело в шишку с бантиком». Формирование происходило по четырем разным единицам измерения: цвета (красный), прочности (твердый), шероховатости (в пупырышке), орнамента (с бантиком). Только набор единиц измерения позволяет адекватно описывать реальные объекты на языке математики … Вот как это выглядит.

Буква «а» с разными индексами указывает на разные единицы измерения. В скобках отмечены единицы измерения, по которым на предварительном этапе выделяется «целое». За скобки выносится единица измерения, по которой формируется множество. В последней строке показан конечный результат — элемент множества. Как видите, если мы используем единицы измерения для формирования набора, то результат не зависит от порядка наших действий. И это уже математика, а не пляски шаманов с бубнами.Шаманы могут «интуитивно» прийти к такому же результату, аргументируя его «доказательствами», ведь единицы измерения не входят в их «научный» арсенал.

Очень просто использовать блоки, чтобы разделить один или объединить несколько наборов в один суперсет. Рассмотрим подробнее алгебру этого процесса.

Обратный косинус и арксинус

Обратный косинус и арксинус

Стандартные триггерные функции являются периодическими, то есть они повторяются.Поэтому одно и то же выходное значение появляется для нескольких входных значений функции. Это делает невозможным построение обратных функций. Для решения уравнений, включающих триггерные функции, необходимо существование обратных функций. Таким образом, математики должны ограничить функцию триггера, чтобы создать эти инверсии.

Чтобы определить обратную функцию, исходная функция должна быть взаимно однозначной . Чтобы существовало однозначное соответствие, (1) каждое значение в домене должно соответствовать ровно одному значению в диапазоне и (2) каждое значение в диапазоне должно соответствовать ровно одному значению в домене. Первое ограничение является общим для всех функций; второй нет. Синусоидальная функция, например, не удовлетворяет второму ограничению, поскольку одному и тому же значению в диапазоне соответствует множество значений в области (см. рис. 1).

Рисунок 1
              Функция синуса не является однозначной.

Для определения обратных функций для синуса и косинуса области определения этих функций ограничены. Ограничение, накладываемое на значения домена функции косинуса, составляет 0 ≤ x ≤ π (см. рис. 2 ).Эта ограниченная функция называется косинусом. Обратите внимание на заглавную «С» в косинусе.

       

Рисунок 2
              График ограниченной функции косинуса.

Функция арккосинуса определяется как функция, обратная ограниченной функции косинуса Cos −1 (cos x ) = x x ≤ π. Следовательно,

         

Рисунок 3
              График функции арккосинуса.

Тождества для косинуса и арккосинуса:

Развитие функции обратного синуса аналогично развитию функции косинуса. Ограничение, накладываемое на значения домена функции синуса, равно

.

Эта ограниченная функция называется синусоидой (см. рис. 4). Обратите внимание на заглавную «S» в слове Sine.

           

Рисунок 4
             График ограниченной синусоидальной функции.

Функция обратного синуса (см. рис. 5) определяется как обратная функция ограниченного синуса y = Sin x

            

Рисунок 5
              График функции обратного синуса.

Следовательно,

Тождества для синуса и обратного синуса:

Графики функций y = Cos x и y = Cos −1 x являются отражениями друг друга относительно прямой y = x . Графики функций y = Sin x и y = Sin −1 x также являются отражением друг друга относительно прямой y = x (см. рис. 6).

Рисунок 6
               Симметрия арксинуса и косинуса.

Пример 1: Используя рисунок 7, найдите точное значение Cos −1 .

         

Рисунок 7
             Чертеж для примера 1.

Таким образом, y = 5π/6 или y = 150°.

Пример 2: Используя рис. 8, найдите точное значение Sin −1 .

      

Рисунок 8
             Чертеж для примера 2.

Таким образом, y = π/4 или y = 45°.

Пример 3: Найдите точное значение cos (Cos −1 0,62).

Использовать тождество косинуса-обратного косинуса:



Обратные триггерные уравнения.

Наверное, это самое сложное и… | Соломон Се | All Math Before College

Вероятно, это самая сложная и сложная тема во всей математике средней школы.

Предварительные условия:

Предварительные условия:

  • Универсальный угол
  • Угол оформления и угол в стандартной позиции
  • Обратные функции
  • Radian & Emage Conversion
  • Признаки функций Trig

просит вас решить функцию Trig:

На самом деле он просит вас использовать обратную функцию триггера навыков.

Самая сложная для понимания часть — это несколько решений. И не все действительны.

напр. sin⁻¹(1/2) означает, что мы знаем, что значение синуса равно 1/2 , и мы хотели бы получить меру угла дуги . Но существует несколько дуговых углов , которые могут иметь одно и то же значение синуса:

  • Против часовой стрелки: 30°, 150°
  • По часовой стрелке: -330°, -210°.

Но чтобы сделать функцию действительной, мы должны сделать ее равной 1-INPUT-1-OUTPUT .
Таким образом, мы должны отфильтровать их и оставить только одно решение, ограничив меру угла .

Мера угла в синусоидальной функции ограничена ДОМЕН , но в обратной функции она стала ДИАПАЗОН .

Путем фильтрации с помощью ДИАПАЗОНА мы должны получить ответ ТОЛЬКО ОДИН И ДОЛЖЕН ТОЛЬКО ОДИН .

Таким образом, после фильтрации всех других решений у нас есть только ОДНО решение, мы называем его:
Принципиальное значение обратной триггерной функции,
, которое появляется в любом калькуляторе , когда вы его вводите.

  • Сначала использовал КАЛЬКУЛЯТОР , чтобы получить основное значение в качестве нашего базового решения .
  • Найдите зеркальное решение каждой тригонометрической функции с помощью Тождеств тригонометрической симметрии :
  • sin(θ) = sin(π - θ) , если θ ПОЛОЖИТЕЛЬНОЕ.
  • sin(θ) = sin(-π - θ) , если θ ОТРИЦАТЕЛЬНО.
  • cos(θ) = cos(-θ)
  • tan(θ) = tan(-π + θ) , если θ ПОЛОЖИТЕЛЬНОЕ.
  • tan(θ) = tan(π + θ) , если θ ОТРИЦАТЕЛЬНО.
  • Добавьте к решению периодичность, чтобы представить все периодические решения: x = θ + 2πn

Пример:

sin(x) = 0,65
  • Используйте калькулятор дуг за 8,7 дуг за 8,7 81388 и получите 5,7 дуг за 8,7938. основное значение: 0,71 Рад .
  • Примените тождество триггера sin(θ) = sin(π-θ) , чтобы получить зеркальное решение: arcsin(0,65) = π - arcsin(0,65) , что будет: 2,43 Rad .
  • С двумя растворами 0.71 Rad и 2.43 Rad и добавим периодичность, мы получим два полных решения:
  • x = 0,71 + 2πn
  • x = 2,43 + 2πn
  • 3 = 9(sin) − 7(sin)

    3 . 0,25

    • Используйте калькулятор , чтобы выполнить arcsin(-0,25) и получить основное значение: -0,25 рад .
    • Примените тождество триггера sin(θ) = sin(-π-θ) , чтобы получить зеркальное решение: arcsin(-0,25) = -π - arcsin(-0,25) , что будет: -2 .89 Рад .
    • с двумя решениями -0.25 RAD и 3.39 RAD и добавить периодичность, мы получим два полных решения:
    • x = -0.25 + 2πn
    • x = -2.89 + 2πn

    Пример:

    cos(x) = −0,7
    • Используйте калькулятор , чтобы вычислить arccos(-0,7) и получить основное значение: 2,35 рад .
    • Примените тождество триггера cos(θ) = cos(-θ) , чтобы получить зеркальное решение: arccos(-0.7) = - arcsin(-0.7) , что будет: -2.35 Rad .
    • с двумя решениями 2,35 RAD и -2. 35 RAD -2.35 RAD и добавить периодичность, мы получим два полных решения:
    • x = 2.35 + 2πn
    • x = -2.35 + 2πn

    Пример :

    cos(x)=0,4
    • Используйте калькулятор , чтобы сделать arccos(0,4) и получить основное значение: 1,16 Рад .
    • Примените тождество триггера cos(θ) = cos(-θ) , чтобы получить зеркальное решение: arccos(0.4) = - arcsin(0.4) , что будет: -1,16 Rad .
    • с двумя решениями 1.16 RAD и -1.16 RAD и добавить периодичность, мы получим два полных решения:
    • x = 1.16 + 2πn
    • x = -1.16 + 2πn

    диапазон различается для каждой функции дуги:

    • Arcsin(x)=θ : -90° < θ < 90°, означает, что угол существует только в Q.1 и Q.4
    • Arccos(x)=θ : 0° < θ < 180° означает, что угол существует только в Q. 1 и Q.2
    • Arctan(x)=θ : -90° < θ < 90°, означает, что угол существует только в Q.1 и Q.4

    Tricks:

    О таких знаках, как 7 и -90 ,
    Просто Думать о
    по часовой стрелке и и против часовой стрелки .

    С специальными триггерными значениями нам вообще не нужен калькулятор, а только для того, чтобы посмотреть на изображение полного единичного круга.Или даже не это, если вы можете это вспомнить.

    Примечание:
    Полная единичная окружность показывает только против часовой стрелки меры угла, что означает ТОЛЬКО ПОЛОЖИТЕЛЬНЫЕ УГЛЫ ,
    поэтому вам нужно сделать свои собственные расчеты, чтобы получить ОТРИЦАТЕЛЬНЫЕ 3УГЛА, также известные как , , ОТРИЦАТЕЛЬНЫХ УГЛОВ, . угол по часовой стрелке измеряет .

    См. YouTube: Оценка обратных тригонометрических функций, Основное введение, Примеры и практические задачи: sin⁻¹(1/2), sin⁻¹(√3/2), [[sin⁻¹(-1/2)] ](https://youtu.be/aRVWs1tDarI?t=4m1s), sin⁻¹(-√3/2), sin⁻¹(0), sin⁻¹(1), sin⁻¹(-1), cos⁻¹(1/2) , cos⁻¹(-√3/2), cos⁻¹(-√2/2), cos⁻¹(0), tan⁻¹(0), tan⁻¹(1), tan⁻¹(-1 ), загар⁻¹(√3), загар⁻¹(-√3/3), просмотреть все.

    Пример: Решите

    sin⁻¹(1/2)
    • Глядя на единичную окружность , мы знаем, что существует несколько мер дуги, которые могут получить синусоидальное значение 1/2 , равное 30°. , 150°, -210°, -330°.
    • Отфильтруйте все остальные по диапазону [-90°, 90°] , мы получим 30° — это ЕДИНСТВЕННЫЙ и ДОЛЖЕН ТОЛЬКО ОДИН ответ.

    Пример: Решите

    sin⁻¹(-1/2)
    • Посмотрите на единичную окружность , мы знаем, что ответ: 210°, 330°, -30°, -150°
    • С фильтрацией по диапазону [-90°, 90°] , поэтому -30° — единственный ответ.

    Пример: Решите

    cos⁻¹(√2/2)

    Как только вы поймете, как решить исходное решение функций, с этим базовым решением будет очень просто.
    Ниже приведены пошаговые решения:

    • Упростите уравнение до sin(θ)=?? Форма .
    • Решите θ с помощью ДВУХ решений в виде θ = ?? + 2πn .
    • Замените θ выражением x и решите уравнение для x .

    Практика хана.

    Пример: Решите 20sin(10x) − 10 = 5

    • Упростите уравнение и получите sin(θ) = 3/4
    • Решите sin(θ) = 3/4 получите решения для 918 : θ = 0,85 + 2πn и θ = 2,29 + 2πn
    • Замените θ выражением x, что дает 10x = 0.85 + 2πn и 10x = 2,29 + 2πn
    • Решив уравнения для x, получим x = 0,085 + 0,2πn и x = 0,229 + 0,2πn
    • 9008com. 3 www.youtube/9008com.3 https://youtube/9008com.3 watch?v=pWdGu9E5nCE

      Функции Acos, Acot, Asin, Atan, Atan2, Cos, Cot, Degrees, Pi, Radians, Sin и Tan в Power Apps — Power Apps

      • Статья
      • 2 минуты на чтение
      Полезна ли эта страница?

      Пожалуйста, оцените свой опыт

      да Нет

      Любая дополнительная обратная связь?

      Отзыв будет отправлен в Microsoft: при нажатии кнопки отправки ваш отзыв будет использован для улучшения продуктов и услуг Microsoft.Политика конфиденциальности.

      Представлять на рассмотрение

      В этой статье

      Вычисляет тригонометрические значения.

      Описание

      Основные функции

      Функция Cos возвращает косинус своего аргумента, т. е. угла, указанного в радианах.

      Функция Cot возвращает котангенс своего аргумента, угол, указанный в радианах.

      Функция Sin возвращает синус своего аргумента, угол, указанный в радианах.

      Функция Tan возвращает тангенс своего аргумента, угол, указанный в радианах.

      Обратные функции

      Функция Acos возвращает арккосинус или арккосинус своего аргумента. Арккосинус — это угол, косинус которого является аргументом. Возвращаемый угол задается в радианах в диапазоне от 0 (ноль) до π.

      Функция Acot возвращает главное значение арккотангенса или арккотангенса своего аргумента.Возвращаемый угол задается в радианах в диапазоне от 0 (ноль) до π.

      Функция Asin возвращает арксинус или арксинус своего аргумента. Арксинус - это угол, синус которого является аргументом. Возвращаемый угол указывается в радианах в диапазоне от -π/2 до π/2.

      Функция Atan возвращает арктангенс или арктангенс своего аргумента. Арктангенс — это угол, тангенс которого является аргументом. Возвращаемый угол указывается в радианах в диапазоне от -π/2 до π/2.

      Функция Atan2 возвращает арктангенс или арктангенс указанных координат x и y в качестве аргументов. Арктангенс — это угол от оси x до линии, содержащей начало координат (0, 0) и точку с координатами ( x , y ). Угол дается в радианах между -π и π, исключая -π. Положительный результат представляет угол против часовой стрелки от оси x ; отрицательный результат представляет угол по часовой стрелке. ATAN2 ( A , B ) равняется ATAN ( B / A ) , за исключением того, что A может равняться 0 (ноль) с функцией ATAN2 .

      Вспомогательные функции

      Функция Degrees преобразует радианы в градусы. π радиан равно 180 градусам.

      Функция Pi возвращает трансцендентное число π, которое начинается с 3,141592...

      .

      Функция радианов преобразует градусы в радианы.

      Примечания

      Если передать этим функциям одно число, возвращаемое значение будет единственным результатом. Если вы передаете таблицу с одним столбцом, содержащую числа, возвращаемое значение представляет собой таблицу результатов с одним столбцом, по одному результату для каждой записи в таблице аргументов. Если у вас есть таблица с несколькими столбцами, вы можете преобразовать ее в таблицу с одним столбцом, как описано при работе с таблицами.

      Если аргумент приводит к неопределенному значению, результатом будет пусто . Это может произойти, например, при использовании обратных функций с аргументами, выходящими за пределы допустимого диапазона.

      Синтаксис

      Основные функции

      COS ( Radians )
      COT ( Radians )
      SIN ( Radians )
      Tan ( Radians )

      • Радиан — обязательно. Угол для работы.

    COS ( Singlecolumntable )
    COT ( Singlecolumntable )
    SIN ( Singlecolumntable )
    Tan ( Singlecolumntable )

    • SingleColumnTable — обязательно.Одностолбцовая таблица углов для работы.

    Обратные функции

    ACOS ( номер )
    ACOT ( номер )
    Asin ( номер )
    ATAN ( номер )

    • Номер - Обязательно. Номер для операции.

ACOS ( SingleColumntable )
ACOT ( Singlecolumntable )
ASIN ( Singlecolumntable )
ATAN ( Singlecolumntable )

  • SingleColumnTable — обязательно. Одностолбцовая таблица чисел для работы.

Атан2 ( X , Y )

  • X — обязательно. X -координата оси.
  • Д - Обязательно. Y -координата оси.

Вспомогательные функции

градусов ( радиан )

  • Радиан — обязательно. Угол в радианах для преобразования в градусы.

Пи ()

Радиан ( Градусов )

  • Градусы - Обязательно.Угол в градусах для преобразования в радианы.

Примеры

Единый номер

Формула Описание Результат
Cos( 1,047197 ) Возвращает косинус 1,047197 радиан или 60 градусов. 0,5
Детская кроватка( Pi()/4 ) Возвращает котангенс 0,785398. .. радиан или 45 градусов. 1
Sin( Pi()/2 ) Возвращает синус 1.570796... радиан или 90 градусов. 1
Тан( Радианы(60) ) Возвращает тангенс 1,047197... радиан или 60 градусов. 1.732050...
Акос( 0,5 ) Возвращает арккосинус 0,5 в радианах. 1.047197...
Акот( 1 ) Возвращает арккотангенс числа 1 в радианах. 0,785398...
Асин( 1 ) Возвращает арксинус числа 1 в радианах. 1.570796...
Атан( 1.732050 ) Возвращает арктангенс числа 1,732050 в радианах. 1.047197...
Атан2( 5, 3 ) Возвращает арктангенс угла от оси x линии, содержащей начало координат (0,0) и координату (5,3), что составляет приблизительно 31 градус. 0,540419...
Атан2( 4, 4 ) Возвращает арктангенс угла от оси x линии, содержащей начало координат (0,0) и координату (4,4), что равно точно π/4 радианам или 45 градусам. 0,785398...
градусов( 1,047197 ) Возвращает количество градусов, эквивалентное 1,047197 радианам. 60
Пи() Возвращает трансцендентное число π. 3.141592...
Радиан ( 15 ) Возвращает эквивалентное число радиан для 15 градусов. 0,261799...

Таблица с одним столбцом

В примерах в этом разделе используется источник данных с именем ValueTable , который содержит следующие данные.Последняя запись в таблице — π/2 радиана или 90 градусов.

9.5 Обратные тригонометрические функции

Тригонометрические функции часто возникают в задачах, и часто это необходимо инвертировать функции, например, чтобы найти угол с заданный синус. Конечно, есть много углов с одним и тем же синусом, поэтому функция синуса на самом деле не имеет обратной, что надежно "отменяет" функцию синуса. Если вы знаете, что $\sin x=0,5$, вы не можете переверните это, чтобы обнаружить $x$, то есть вы не можете решить для $x$, так как существует бесконечно много углов с синусом $0.5$. Тем не менее, полезно иметь что-то вроде обратного к синус, пусть и несовершенный. Обычный подход состоит в том, чтобы выбрать несколько набор углов, которые производят все возможные значения синуса ровно один раз. Если мы «отбросим» все остальные углы, то в результате функция имеет правильную обратную.

Синус принимает все значения от $-1$ до $1$ ровно один раз на интервал $[-\pi/2,\pi/2]$. Если мы обрежем синус, оставив только интервал $[-\pi/2,\pi/2]$, как показано на рисунке 9.3}=х$. Этот не работает с синусом и "обратным синусом", потому что обратный синус - это функция, обратная усеченному синусу, а не функция реального синуса. Верно, что $\sin(\arcsin(x))=x$, т. е. синус отменяет арксинус. Неправда, что арксинус отменяет синуса, например, $\sin(5\pi/6)=1/2$ и $\arcsin(1/2)=\pi/6$, поэтому выполнение сначала синуса, а затем арксинуса не возвращает нас туда, где мы началось. Это потому, что $5\pi/6$ не находится в домене усеченный синус. Если мы начнем с угла между $-\pi/2$ и $\pi/2$ тогда арксинус обращает синус: $\sin(\pi/6)=1/2$ и $\arcsin(1/2)=\pi/6$.

Чему равна производная арксинуса? Так как это инверсия функцию, мы можем найти производную, используя неявное дифференциация. Предположим, что $y=\arcsin(x)$. потом $$\sin(y)=\sin(\arcsin(x))=x.$$ Теперь, взяв производную от обеих частей, получим $$\выравнивание{ y'\cos y &= 1\cr y'={1\over \cos y}\cr }$$ Как и следовало ожидать, при использовании неявного дифференцирования $y$ появляется на правая сторона здесь. Мы определенно предпочли бы, чтобы $y'$ было написано в терминах $x$, и, как и в случае $\ln x$, мы действительно можем сделать это здесь.2 }}.$$ Обратите внимание, что усеченный косинус использует другой интервал, чем усеченный синус, поэтому что если $y=\arccos(x)$, мы знаем, что $0\le y\le \pi$.

Добавить комментарий

Ваш адрес email не будет опубликован. Обязательные поля помечены *